Вы находитесь на странице: 1из 141

DNB Mock Test 1

DNB MOCK TEST

Marking: +1 Marks for correct answer and no negative Marking for incorrect answer.
Test Duration (mins) :180
Time Left (mins) : 180
Instructions
Once the test has started, do not press the refresh button (or F5 on your keyboard)
It is advisable to save the test regularly to avoid
losing your information, save test refers to storing the attempted part of the test.
In case of accidental failure of internet connectivity the system will save the attempted
portion of the test automatically.
Once the test time is over, you will be awarded a grace duration to wrap up the test and
"Submit" it.
Submit test refers to the final completion of test; once you submit the
test you will not be able to edit / preview your answers.
Please ensure that you are connected to the internet, while submitting the test.
DNB 7 MOCK TEST

(Q.1) Which of the following is false?

(a) Bcl-6-burkitts lymphoma

(b) Bcl-2-follicular lymphoma

(c) CD-10 mantle cell lymphoma

(d) CD 34 in DLBCL

Your Response :
Correct Answer : D

Exp: CD 34 in DLBCL

bcl-6 positivity common in Burkitt's. Burkitt’s lymphoma is also positive for IHC
(immunohistochemical) marker

CD10 and Ig.

bcl-2 -most commonly used marker for Follicular lymphoma though ,CD10 is usua
expressed by mantle cell lymphoma...but, Aberrant expression of marker CD-10 is
common for mantle cell lymphoma but,CD34 is not expressed by DLBCL

(Q.2) A benign tumour in the pterygoid canal would spare which of the following nerve fibers
(a) Postganglionic parasympathetic fibers

(b) Taste fibers from the palate

(c) Postganglionic sympathetic fibers

(d) General visceral afferent (GVA) fibers

Your Response :
Correct Answer : A

Exp: Postganglionic parasympathetic fibers

The nerve of the pterygoid canal (Vidian nerve), formed by the junction of the gre
(superficial)petrosal nerve and the deep petrosal nerve in the cartilaginous substa
which fills the foramen lacerum, passes forward, through the pterygoid canal, and
the pterygopalatine ganglion.

Greater petrosal nerve carries taste fibres from palate as well


as preganglionicsecretomotor(parasympathetic) fibres towards the pterygopalati
ganglion

Deep petrosal nerve carries the postganglionic sympathetic fibres from T1 spinal s
(superior cervical ganglion)

(Q.3) If rapidly progressive cancers are missed by a screening test, which type of bias will occur?

(a) Lead-time bias

(b) Length bias

(c) Selection bias

(d) Surveillance bias

Your Response :
Correct Answer : B

Exp: Length bias

Park, 20th Edition, Page no 69

Rapidly progressive cancers will be less likely to be detected by a screening test if


symptoms rapidly develop because the window period between the time the canc
be detected when it is asymptomatic by a screening test and the time it will becom
clinically apparent is short. This is described as length bias. Screening tests will yie
outcomes because they detect more slowly growing tumors. Lead-time bias occur
the screening test advances the time of diagnosis, but no true prolongation of life
because survival for women who are screened and those who are not is the same
the time the cancer occurs. Information bias occurs when there is a systematic dif
in the way data are collected (inaccurate or imprecise measure) for either the exp
the outcome. Recall bias is one form of information bias. Selection bias occurs wh
inclusion of a subject in a study group is linked to the exposure of interest. As an e
for a case-control study, if women who use oral contraceptives are suspected mo
of having deep vein thrombosis (DVT), they would be hospitalized more often for
evaluation and diagnosed more often than controls. Selection bias can also occur
studies and is related to differential loss to follow-up. Surveillance bias refers to
overdetection of the disease of interest because one of the groups goes to the do
has a diagnostic test) more often than does another group. For example, women
postmenopausal estrogens presumably go to the doctor (and probably have
mammograms) more frequently than women who do not; thus, women who take
estrogens may be more likely to have breast cancers detected because of the incr
surveillance.

(Q.4) Excessive contraction of following muscle causes dislocation of jaw

(a) . Temporalis

(b) . Lateral pterygoid

(c) . Medial pterygoid

(d) . Zygomaticus Major

Your Response :
Correct Answer : B

Exp: Lateral pterygoid

Contraction of the lateral pterygoid acts to pull the disc and condyle forward with
glenoid fossa and down the articular eminence; thus, action of this muscle serves
the mouth & cause protrusion of lower jaw.

The most common disorder of the TMJ is disc displacement (dislocation). In essen
is when the articular disc, attached anteriorly to the superior head of the lateral p
muscle and posteriorly to the retrodiscal tissue, moves out from between the con
the fossa, so that the mandible and temporal bone contact is made on something
than the articular disc.

(Q.5) All is true about stapes except


(a) Derivative of 2nd pharyngeal arch

(b) Covers the oval window

(c) Controlled by stapedius to protect middle ear

(d) Damage to stapedius produces hyperacusis

Your Response :
Correct Answer : C

Exp: Controlled by stapedius to protect middle ear

The stapedius pulls out oval window from the fossa ovalis, if there is loud sound, a
this protectivemove is called stapedial reflex. This prevents damage to the inner

If stapedius is damaged, the patient suffers from hyperacusis, due to absence of s


reflex

2nd arch give stapes, syloid and upper hyoid.

Pharyngeal Arch Artery Cranial Nerve Skeletal elements Muscles


Arch

1 Terminal Branch Maxillary and Derived from arch Muscles of


of maxillary mandibular cartilages (originating mastication
artery division of from neural crest): (temporalis
trigemenial (V) masseter, a
From maxillary pterygoids)
cartilages: mylohyoid,
Alispenoid, incus belly of dig
tensor tym
From mandibular: tensor veli
(originate f
Mackel’s cartilage, cranial som
malleus 4)
Upper portion of
external ear (auricle)
is derived from
dorsal aspect of
1stpharyngeal arch.

Derived by direct
ossification from arch
dermal mesenchyme:
Maxilla, zygomatic,
squamous portion of
temporal bone,
mandible

2 Stapedius artery Facial nerve (VII) Stapes, styloid Muscles of


(embryologic) process, stylohyoid expression
and ligament, lesser (orbicularis
cortiotympanic horns and upper rim orbicularis
artery (adult) of hyoid (derived auricularis,
from the second arch platysma, f
cartilage; originate ooccipitalis
from neural crest). buccinator
posterior b
Lower portion of digastric,
external ear (auricle) stylohyoid,
is derived from stapedius (
2nd pharyngeal arch. from crania
somitomer

3 Common carotid Glossopharyngeal Lower rim and Sytlophary


artery, most of (IX) greater horn of hyoid (originate f
internal carotid (derived from the cranial som
third arch cartilage; 7)
originate from neural
crest cells)

4 Left: Arch of Superior Laryngeal cartilages Constrictor


aorta; laryngeal branch (Derived from the pharynx,
th
of vagus (X) 4 arch cartilage, cricothyroi
Right: Right originate from lateral levator veli
subclavian artery; plate mesoderm) (originate f
Original sprouts occipital so
of pulmonary 4)
arteries

6 Ductus Recurrent Laryngeal cartilages Intrinsic mu


arteriosus; roots laryngeal branch (derived from the 6th- larynx (orig
of definitive of vagus (X) arch cartilage; from occip
pulmonary originate from lateral somites 1 a
arteries plate mesoderm)

(Q.6) Which of the following statements is false regarding subclavian steal syndrome?
(a) . Results from thrombosis of the left subclavian artery distal to the vertebral a

(b) Reversal of blood flow in ipsilateral vertebral artery

(c) . Blood reaches left vertebral artery via carotid circulation

(d) Vertigo occurs due to vertebro-basilar insufficiency

Your Response :
Correct Answer : A

Exp: Results from thrombosis of the left subclavian artery distal to the vertebral arte

Subclavian steal steno-occlusive disease, is a constellation of signs and symptom


arise fromretrograde (reversed) vertebral artery (blood) flow or retrograde intern
thoracic artery flow due to a proximal subclavian artery stenosis (narrowing) and/
occlusion.

SSS results when the short low resistance path (along the subclavian artery) becom
a high resistance path (due to narrowing) and blood flows around the narrowing v
arteries that supply the brain (left and right vertebral artery, left and right interna
artery). The blood flow from the brain to the upper limb in SSS is considered to
be stolen as it is blood flow the brain must do without.

Normally, blood flow from the aorta into the subclavian artery and then some of t
blood leaves via the vertebral artery to supply the brain.

In SSS a reduced quantity of blood flows through the proximal subclavian artery. A
result, blood travels up one of the other blood vessels to the brain (the other vert
the carotids) goes around the cerebral arterial circle (of Willis) and via the (ipsilate
vertebral artery to the subclavian (with the proximal blockage) and feeds blood to
the distal subclavian artery (which supplies the upper limb and shoulder).

(Q.7) The vertebral artery traverses all of the following except?

(a) Foramen magnum

(b) Subarachnoid space

(c) Intervertebral foramen

(d) Foramen transversarium

Your Response :
Correct Answer : C
Exp: Intervertebral foramen

Between every pair of vertebræ are two apertures, the intervertebral foramina, fo
passage of the spinal nerves and vessels.

They can be occluded by arthritic degenerative changes and space-occupying lesio


tumors, metastases and spinal disc herniations.

(Q.8) Which of the following pathway is involved in the ability to recognize an unseen familiar object placed
hand?

(a) Dorsal spinocerebellar tract

(b) Anterior spinothalamic tract

(c) Dorsal column

(d) Posterior spinothalamic tract

Your Response :
Correct Answer : C

Exp: Dorsal column

The ability to recognize an unseen familiar object placed in the hand by feeling fo
shape & texture is stereognosis carried by the dorsal column-medial lemniscal sys

(Q.9) Transfer of a donor chromosome fragment by a temperate bacterial virus is defined as?
(a) Competence
(b) Conjugation
(c) Recombination
(d) Transduction
Your Response :
Correct Answer : D
Exp: Transduction
Anant Narayan 7th Edition, Page 56

Transformation, transduction, and conjugation are critical processes in which DNA


transferred from one bacterium to another. Transformation, the passage of high-
molecular-weight DNA from pne bacterium to another, was first observed in
pneumococci. Later studies have shown that, at least in S. pneumoniae, double-st
DNA is “nicked” by a membrane- bound endonuclease, initiating DNA entry into t
cell. One of the nicked DNA strands is digested, and the other is integrated into th
genome. Transduction, which can affect many bacteria, is a process in which a fra
donor chromosome is carried to a recipient cell by a temper ate virus (bacterioph
generalized transduction, the phage virus can carry any segment of the donor
chromosome; in restricted transduction, the phage carries only those chromosom
segments immediately adjacent to the site of prophage attachment. In conjugatio
DNA is passed from one bacterium to another. However, instead of the transfer o
DNA, a small loop of DNA, called a plasmid, is passed between cells. Examples of p
are the sex factors and the resistance (R) factors.

(Q.10) All is true about Brown Séquard syndrome except

(a) Hemisection of spinal cord

(b) Ipsilateral loss of vibration sensations

(c) Ipsilateral loss of crude touch sensations

(d) Ipsilateral paralysis below the level of lesion

Your Response :
Correct Answer : C

Exp: Ipsilateral loss of crude touch sensations

Crude touch is carried contra-laterally by anterior spinothalamic tract

Hence, in BS syndrome (hemisection of spinal cord) the loss of crude touch is co


lateral

Brown-Séquard syndrome, also known as Brown-Séquard's hemiplegia, is a loss


motoricity (paralysis and ataxia) and sensation caused by the lateralhemisection
spinal cord.

The hemisection of the cord results in a lesion of each of the three main neural s
the principal upper motor neuron pathway of the corticospinal tract, one or both
columns and in the spinothalamic tract.

As a result of the injury to these three main brain pathways the patient will prese
three lesions.

The corticospinal lesion produces spastic paralysis on the same side of the body
of moderation by the UMN).

The lesion to fasciculus gracilis or fasciculus cuneus results in ipsilateral loss of vi


and proprioception (position sense).

The loss of the spinothalamic tract leads to pain and temperature sensation bein
from the contralateral side beginning one or two segments below the lesion.

At the lesion site all sensory modalities are lost on the same side, and also an ips
flaccid paralysis.

(Q.11) The inferior tibio-fibular joint is what type?

(a) Synchondrosis

(b) Syndesmosis

(c) Gliding Synovial joint

(d) Plane synovial

Your Response :
Correct Answer : B

Exp: Syndesmosis

TYPES OF JOINTS IN BODY : HEAD-TO-TOE :

JOINT TYPE

Temporomandibular joints Condylar (Bicondylar) synovial joint

Cricothyroid and cricoareytenoid joint Synovial joint

Atlanto-occipital joint, Wrist joint and Ellipsoid joint


Metacarpo-phalangeal joint

Sternoclavicular joint Saddle (sellar) joint

Ear ossicles Saddle joint

Melleo-Incal joint Ball and socket joint

Incudo-stapedial joint Syndesmosis

Stapes footplate

Joint between ala of vomer and Schindylesis (Wedge and groove


Rostrum of sphenoid suture)

Costo-vertebral joint Plane synovial joint

Costo-transverse joint Synovial joint


1st chondrosternal joint, Primary cartilaginous joint /
Synchondrosis / Hyaline cartilaginous
All Costochondral joints, joint.
Spheno-occipital joint

Joint between epiphysis and diaphysis


of growing bone

2nd – 7th chondrosternal joint Synovial joint

5th – 9th costal cartilage articulation Synovial joint

10th costal cartilage is united to 9thcostal Fibrous tissue.


cartilage by

Superior and inferior Radio-ulnar joints Pivot (Trochoid) joint

Median Atlanto-axial joint

Middle radio-ulnar joint Syndesmosis

Sterno-clavicular joint Saddle joint

Elbow and Ankle joint Hinge synovial joint

Hip joint Ball and socket synovial joint

Knee joint Compound synovial joint

2 condylar joints between medial and


lateral condyles of femur and tibia.

1 saddle joint between femur and


patella.

Superior Tibio-fibular joint Plane synovial joint

Middle Tibio-fibular joint Fibrous joint

Inferior Tibio-fibular joint Syndesmosis`

Talo-calcaneal / Subtalar joint Plane synovial joint

Calcaneo-cuboid joint Saddle joint

Talo-calcaneo-navicular joint Ball and socket synovial joint


Other intertarsal joints Plane synovial joint

Symphysis pubis Secondary cartilaginous joint /


Symphyses / Fibrocartilagenous joint
Sacro-coccygeal joint

Manubriosternal joint

Intervertebral joints

(Q.12) Radiation can act in all the following ways EXCEPT:


(a) Autoimmunity
(b) Free radical injury
(c) Mutation
(d) DNA breakage
Your Response :
Correct Answer : A
Exp: (Autoimmunity)
Cells are directly killed or are affected by radiation to prevent mitosis. The local b
supply is damaged causing cell deaths. Damaged cells due to free radical injury b
radiation become victims of tissue defense systems.
(Q.13) First dermatological sign after radiation treatment is:
(a) Erythema

(b) Necrosis

(c) Burns

(d) Deep ulcer

Your Response :
Correct Answer : A

Exp: (Erythema)

Skin reaction can be seen with two weeks of fractionated radiotherapy (RT).

Erythema is the earliest observed feature followed by desquamation (dry/moist)

Chronic reaction can be seen starting at 6-12 months after irradiation.

Atrophic, easily damageable epidermis, hyperpigmentation, thin skin with hair lo


occurs. Rarely malignant change can occur.
(Q.14) Extent of cardiotoxicity of chemotherapy and radiotherapy is best diagnosed by:
(a) Echocardiogram

(b) ECG

(c) Radionuclide Scan

(d) Endomyocardial biopsy

Your Response :
Correct Answer : D

Exp: (Endomyocardial biopsy)

Radiation induced necrosis and similar changes are best assessed by biopsy of
the particular affected tissue of organ.
(Q.15) Which antiviral agent most often causes anemia, leukopenia, or thrombocytopenia?

(a) Ribavirin

(b) Zidovudine

(c) Amantadine

(d) Acyclovir

Your Response :
Correct Answer : B

Exp: Zidovudine

Harrison’s 17th Edition, Page no 1179


The most serious side effect of zidovudine is bone marrow suppression. Ganciclo
(Cytovene) may also cause these adverse effects, but acyclovir (Zovirax) usually d
Didanosine and zalcitabine (other dideoxynucleosides used for HIV) are less likely
cause bone marrow suppression than zidovudine and are more frequently associ
with pancreatitis and sensorimotor neuropathy.

(Q.16) “Tectal beaking” is MRI feature of:


(a) Dandy Walker malformation
(b) Arnold-Chiari malformation
(c) Aqueductal stenosis
(d) Third ventricular tumor
Your Response :
Correct Answer : B
Exp: (Arnold-Chiari malformation)
Arnold Chiari I malformation
Chiari I malformation (cerebellar ectopia and seen in adulthood)
Definition: - Descent of otherwise normal cerebellar hemispheres below the forame
magnum, usually involving tonsils.
Features: Peg like, pointed tonsils displaced into upper cervical canal
 0–10 years ◊ 6 mm at least
 10–30 years ◊ 5 mm
 30–40 years ◊ 4 mm
Associated anomalies: Syringomyelia (30–60%), hydrocephalus (20–25%), basilar
invaginations (25–50%), Klippel-Feil syndrome (5–10%) and atlantooccipital assimila
5%)
Chiari II malformation (childhood)
Definition: Descent of dysplastic cerebellar hemisphere below the foramen magnum
involving inferior vermis, which is everted instead being inverted, causing 4th ventri
reduce to a coronal cleft and medulla oblongata to elongate and kink.
Features:
---------------------------------------------------------------------------------------------------------------
Skull and Dura Brain Ventricles Spine and Cord:
---------------------------------------------------------------------------------------------------------------
Calvarial defects (Lacunar skull Inferior displaced Hydrocephalus
(90%) Myelomeningcele (100%)
Luckenschadel) vermis Batwing frontal horns Syringohydromyelia
Small posterior fossa Medullary spurs/kink Elongated tube-like
4th vent. Diastematomyelia
Beaked tectum Fenestrated falx Incomplete/absent C1 arch
Heart-shaped incisura Interdigitated gyri Large mass intermedia
Hypoplastic tentorium Towering cerebellum Colpocephaly
Gaping foramen magnum Polymicrogyria
Concave clivus and petrous ridge Callosal agenesis Heterotopias
---------------------------------------------------------------------------------------------------------------
Chiari type III and IV are extremely rare.
(Q.17) Egg shell calcification in lymph nodes can be caused
by all except ?
(a) Sarcoidosis
(b) Silicosis
(c) Tuberculosis
(d) Leukemia
Your
Response
:
Correct D
Answer :
Exp: Leukemia
Egg Shell calcification in lymphnodes :
Silicosis
Sarcoidosis
Coal workers pneumoconiosis
Lymphoma following radiotherapy
Histoplasmosis
Progressive massive fibrosis
TB
Coccidiomycosis

(Q.18) Radiosensitizing substances include:


(a) Oxygen

(b) SR 2508

(c) Metronidazole

(d) All of the above

Your Response :
Correct Answer : D

Exp: (All of the above)

 Hypoxic cells are less radiosensitive


 Ratio of dose for anoxic cells to that for well-oxygenated cells is called the oxyg
enhancement ratio
 Augmentation of oxygen is the basis for radiation sensitization
 Metronidazole is a radiosensitizer
 Sulphydryl compounds interfere with free radical generations and may act as r
protectors
 Compounds that incorporate into DNA and alter its stereochemistry like cispla
halogenated pyrimidines, compounds that inhibit DNA synthesis like hydroxyure
compounds that deplete thiols, buthionine sulfoximine, augment radiation effec
also a radiosensitizer.
 Other factors influencing it are tumor mass growth rate and cell cycle phase.
(Q.19) "Bracket-like” intracranial calcification is seen in:
(a) Sturge Weber syndrome
(b) Oligodendroglioma
(c) Corpus callosal lipoma
(d) Tuberous sclerosis
Your
Response :
Correct
C
Answer :
Exp: “Bracket calcification” on X-ray skul is
diagnostic of lipoma of corpus callosum.

(Q.20) Maximum permissible dose equivalent of radiations recommended by NCRP for pregnant women is:
(a) 0.1 rem
(b) 0.5 rem
(c) 1 rem
(d) 5 rem
Your Response :
Correct Answer : B
Exp: (0.5 rem)
The NCRP recommends a total dose equivalent limit (excluding medical exposure
rem (50 mSv) for the embryo, fetus. Once a pregnancy become known, exposure
embryo-fetus should be no greater than 0.05 rem (0.5 mSv) in any month (exclud
medical exposure).
NCRP Recommendation of radiation dose limits:
-----------------------------------------------------------------------------------------------------------
Class of exposed individual Rems mSv
-----------------------------------------------------------------------------------------------------------
1) Occupational exposures (annual)

Stochastic effects 5 50

Non-stochastic effects

Lens of eye 15 150

All other areas (e.g., red marrow, 50 500

breast, lung, gonads, skin, and extremities)

Lifetime cumulative exposure 1 (x age in year 10 (x age in year

2) Public exposure (annual)

Effective dose equivalent limit 0.5 5

Dose equivalent limits for lens of eye, skin,5 50

and extremities
3) Trainees under 18 years of age

Effective dose equivalent limit 0.1 1

Dose equivalent limit for lens of eye, 5 50

skin, and extremities

4) Embryo-fetus exposures

Total dose equivalent limit 0.5 5

Dose equivalent limit in a month 0.05 0.5

-----------------------------------------------------------------------------------------------------------
- Excluding background and medical exposures, but including both internal and
external exposures
- Rem = rads x quality factor (R)
- Because the quality factor for X-rays is 1, hence rad = rem
- In fact at diagnostic energy level the rad, rem and roentgen usually will be co
(Q.21) “CHART” compared with conventional radiotherapy gives a significant improvement in survival of pat
with:
(a) Mycosis fungoides

(b) Diffuse Non-Hodgkin’s lymphoma

(c) Aggressive head and neck malignancies

(d) Non-Small Cell Lung Cancer

Your Response :
Correct Answer : D

Exp: CHART is Continuous Hyperfractionated Acclerated Radiotherapy, best suitable f


management of NSCLC (Non-Small Cell Lung Cancer). Compared with convention
radiotherapy CHART gives a significant improvement in survival of these patients

(Q.22) Which of the following local anaesthetic is sympathomimetic:


(a) Cocaine

(b) Procaine

(c) Lignocaine

(d) Tetracaine
Your Response
:
Correct Answer A
:
Exp: Cocaine

• Cocaine is a sympathomimetic local anaesthetic and thus


adrenaline should not be added to it.

(Q.23) All of the following features distinguish infant larynx from adult EXCEPT:

(a) Epiglottis is long and leaf

(b) Subglottic region is narrowest laryngeal portion

(c) Large tongue

(d) Lower placement of the larynx

Your Response :
Correct Answer : D

Exp: Lower placement of the larynx

• Following features of infant larynx in contrast to adult larynx:

1. Epiglottis is long and leafy.

2. Subglottic region is the narrowest portion of larynx.

3. Larynx is placed at a higher level (in adults, it is placed at the level of C3-C6
vertebrae).

Anatomic Differences between the Child's and Adult's Larynx

Child's larynx Adult's larynx

Size Smaller Larger

Shape Lumen is funnel-shaped with the narrowest part Narrowest part of


below the vocal cords and within the cricoid ring lumen is at the vo
cords

Location Higher, closer to the tongue base; vertical extent Vertical extent is l
is opposite C3, C4, C5 vertebrae; more anterior opposite C4, C5, C
vertebrae

Epiglottis Longer, narrower, and "U" shaped; the angle Shorter and wider
between glottis and epiglottis is more acute;
increased chance of airway obstruction (see
Figure 1-8)

Vocal cords Angled in relation to the axis of trachea; shorter; Perpendicular to t


more cartilaginous; more distensible; more likely axis of trachea
to be injured

Rigidity The laryngeal cartilages are softer and more More rigid
pliable

Response Mucous membrane is more loosely attached and Less vulnerable to


to trauma swells more readily when traumatized or trauma and infect
infected

(Q.24) Composition of sodalime is which is the correct:


NaOH Ca(OH)2 KOH

(a) .5% 94% 1%

(b) .20% 80% -

(c) .80% 20% -

(d) .15% 84% 1%

Your
Response :
Correct A
Answer :
Exp: Soda-lime is a mixture of 94% calcium hydroxide with 1%
potassium hydroxide and 5% sodium hydroxide.

(Q.25) After contrast media injection in radiology department a patient develop severe hypotension, bronch
and cyanosis. Which of the following should be used for treatment:

(a) Atropine

(b) Aminophylline
(c) Dopamine

(d) Adrenaline

Your Response :
Correct Answer : D

Exp: Adrenaline

Adrenaline is drug of choice for severe anaphylaxis. Concentrations used should


1:10,000 if to be given IV or 1:1,000 if to be given SC.

(Q.26) A 38 year old man is posted for extraction of last molar tooth under general anaesthesia as a day case
wishes to resume his work after 6 hours. Which one of the following Induction agents is preferred:

(a) Thiopentone sodium

(b) Ketamine

(c) Diazepam

(d) Propofol

Your Response :
Correct Answer : D

Exp: Propofol is the choice of agent in the day care surgery.

(Q.27) Regarding the rebreathing prevention valve which of the following is incorrect:

(a) Should be as far as possible from patient

(b) Should be light

(c) Suitably designed.

(d) Installed at the expiratory end of the tube

Your Response :
Correct Answer : A

Exp: Should be as far as possible from patient

Rebreathing prevention valve is also called as expiratory valve or more commonl


by the name adjustable pressure limiting valve (APL valve). It should be as near t
patient to permit maximum exhalation of expired gases. The most commonly use
is of Heidbrink type.

(Q.28) All action occur in mitosis EXCEPT

(a) Cytokinesis

(b) Chromatids separate

(c) DNA replication

(d) The kinetochore becomes evident

Your Response :
Correct Answer : C

Exp: DNA replication

Ganong, 22nd Edition, Page no 21


DNA replication or DNA synthesis occurs during the S phase of the cell cycle. G1 S
make up interphase. Mitosis completes the cell cycle and is composed of propha
metaphase, anaphase, and telophase. Cytokinesis is the division of the cell cytop
that occurs in telophase and is orchestrated by the microfilaments. Chromatids s
and move toward the poles of the cell in anaphase. The nuclear membrane and n
disappear during metaphase.

(Q.29) Pin index for nitrous oxide is:

(a) 2, 5

(b) 3, 5

(c) 1, 5

(d) 2, 6

Your
Response
:
Correct B
Answer :
Exp: 3, 5

Pin index for different gases are:

Oxygen 2, 5
Air 1, 5

Nitrous oxide 3, 5

Cyclopropane 3, 6

Nitrogen 1, 4

Entonox ( 50% oxygen +50% nitrous


oxide) 7

Carbon dioxide 2, 6.

(Q.30) First spinal anaesthesia was given by:

(a) August Bier

(b) Sicard

(c) Morton

(d) Priestley

Your Response :
Correct Answer : A

Exp: August Bier

Although first spinal anaesthesia in dogs was given by Leonard Corning but first s
human beings was given by August Bier in 1898.

(Q.31) During laryngoscopy and intubation which of the maneuver is not performed:

(a) Flexion of neck

(b) Extension of head at atlanto-occipital joint

(c) The laryngoscope is lifted upwards levering Lower the upper incisors

(d) . In straight blade laryngoscope the epiglottis is lifted by lip

Your Response :
Correct Answer : C

Exp: The laryngoscope is lifted upwards levering Lower the upper incisors

Laryngoscopy requires flexion at cervical spine and extension at atlanto-occipital


and this can be An achieved by putting a 6-8 em pillow below the occiput.

Macintosh curved blade draws the epiglottis forward ain from vallecula while stra
blade lifts the epiglottis (because epiglottis in newborn is large and leafy).

Upper incisors are most vulnerable to damage by laryngoscopy so laryngoscope s


not be levered against them.

(Q.32) Both hepatic and renal failures can be caused by which of the following:

(a) Paracetamol toxicity

(b) Carbon tetrachloride

(c) Arsenic

(d) Copper sulphate

Your
Response :
Correct D
Answer :
Exp: Copper sulphate (PCM, CCl4, AgNO3 - liver failure. As -
kidney failure. CuSO4- both hepatic & Renal failure.)

(Q.33) Sodium fluoride may be used for preservation of blood for detection of :

(a) Cyanide

(b) Arsenic

(c) Alcohol

(d) Urine

Your Response :
Correct Answer : C

Exp: (Preservatives used in poisoning cases & Rabies-

1) All poisoning cases- saturated soln. of common salt, Acid poisoning-


rectified spirit (95%ethanol + 5% methanol), EXCEPT- Carbolic acid- Thymol.

2) Blood for Alcohol- NaF.

3) Urine- Thymol. Faeces- rectified spirit.


4) CO, HCN- liquid paraffin or no preservative if fresh.

5) As, Sb- 15 cm long bone.

6) Rabies/virological specimens- 50% glycerol saline, keep hippocampus,


cerebral cortex, cerebellum, medulla.)

(Q.34) Acrodynia is seen in:

(a) Lead poisoning

(b) Mercury poisoning

(c) Cyanide poisoning

(d) Tin poisoning

Your Response
:
Correct Answer B
:
Exp: (Acrodynia = Pink disease = Feer’s disease = Swift syndrome-
painful & pink extremities, is seen in Mercury poisoning.)

(Q.35) Green coloured urine is seen after ingestion of:

(a) Copper sulphate

(b) Phenol

(c) Organophosphorus

(d) Cyanide

Your
Response
:
Correct B
Answer :
Exp: (Carboluria = green urine, is seen in
Phenol (Carbolic acid) poisoning.)

(Q.36) Dibucaine number is utilized for diagnosing:

(a) Atypical pseudocholinesterase


(b) Dual block

(c) Degree and intensity of block after succinylcholine

(d) Duration of block


Your Response :
Correct Answer : A
Exp: Atypical pseudocholinesterase
Atypical pseudocholinesterase is a genetic disease in which the patients
pseudocholinesterase can not metabolize succinylcholine and there can be very
prolonged block.

Dibucaine a local anaesthetic can inhibit 80% of normal enzyme and 20% of abno
enzyme. So normal dibucaine number is 80.

(Q.37) The Non-poisonous salt of Cyanide is:

(a) Potassium Cyanide

(b) Hydrocyanic acid

(c) Sodium Cyanide

(d) Potassium ferrocyanide

Your Response :
Correct Answer : D

Exp: (Poison used as a treatment of another poison- (1) CuSO4(0.2%) for P4, (2) 1% Po
ferrocyanide for CuSO4, (3) Ethyl for methyl alcohol, (4) Atropine for Organopho
poisoning.)

(Q.38) Artificial bruises are made using:

(a) Calotropis

(b) Ricinus communis

(c) Semecarpus Anacardium

(d) a and c

Your Response :
Correct Answer : D
Exp: (Artificial bruises are made using the juice of Calotropis plant & the juice of Sem
anacardium (Marking nut). Artificial bruises are found only on parts accessible b
hands, they have sharp margins, show signs of inflammation like itching, vesicati
redness, swelling, as against a True bruise which may be found anywhere on the
has irregular margins, shows no signs of inflammation except swelling, redness.)

(Q.39) Not a clinical feature of malignant hyperthermia:

(a) Decreased end tidal carbon dioxide

(b) Metabolic acidosis

(c) Hyperkalemia

(d) Increased creatine kinase


Your Response :
Correct Answer : A
Exp: Decreased end tidal carbon dioxide
KDT, 6th Edition, Page no 372

In malignant hyperthermia the end tidal CO2 may rise to more than 100 mmHg a
temperature may rise to >109oF. Usual cause of death is severe metabolic acidos

(Q.40) All are true EXCEPT:

(a) Nux vomica contain strychnine

(b) Cantharides may cause priapism

(c) Sui-picking involves Ricinus communis

(d) Acrodynia is caused by mercury poisoning

Your Response :
Correct Answer : C

Exp: ('Suis' involve- Abrus + Datura + Opium + Onion. All these are mixed, rolled into
or ‘suis’ and dried in the Sun and then are used to rob a passenger or to kill an an

(Q.41) ‘Marsh’s test’ is used in …. poisoning:

(a) Arsenic

(b) Mercury
(c) Zinc

(d) Lead

Your
Response :
Correct A
Answer :
Exp: (Tests used to detect Arsenic-

(1) Marsh’s test- Hydrogen generator + Arsenic compound


→ bluish-green/violet/purple flame & garlic odour.

(2) Reinsch’s test- Specimen + pure HCl → boiled with Cu


strip → coated steel-grey/ black.)

(Q.42) Indole ring is present in

(a) Tryptophan

(b) Valine

(c) Methionine

(d) Histidine

Your
Response :
Correct A
Answer :
Exp: (Tryptophan)

Tryptophan ◊Aromatic ring containing amino


acid: - 2-amino-3 (3-Indonyl) propanoic acid

• Side chain containing sulfur Atoms — Amino


acids: - eg. cysteine and Methionine.

(Q.43) Leucine zipper; motif is a mediator for

(a) Membrane attack complexes

(b) Ligand membranes

(c) Binding of regulatory proteins to DNA


(d) Cyclic GMP

Your
Response :
Correct C
Answer :
Exp: (Binding of regulatory proteins of DNA)

Several motifs mediate the binding of regulatory


protein to DNA : three unique motifs —

1. Helix-Turn-Helix motif

2. Zinc-finger motif

3. Leucine-zipper motif

• These are accounts for many of these specific


protein-DNA interaction

• The protein-DNA interaction are maintained by


Hydrogen bonds and van-der-waals forces.

(Q.44) Multienzyme complex in humans

(a) Fatty acid synthetase

(b) Malonyl CoA Carboxylase

(c) Carbamoyl phosphate synthetase

(d) Adenosine phospho ribosyl transferase

Your Response :
Correct Answer : A

Exp: (Fatty acid synthetase)

• Fatty acid synthase complex

(i) Is a polypeptide containing seven enzymes activites = multienzyme complex

(ii) The complex is a dimer of two identical polypeptide monomers, each consisti
seven enzymes and Acyl carrier protein (ACP) * Head to tail arrangement of two
monomers.

(iii) Enzymes are ◊ Thioesterase, Ketoacyl Reductase, Enoyl reductase, Hydratase


Malonyl transacylase, Acetyl transacylase, Ketoacyl synthase.

• Acetyl CoA carboxylase ◊ is a Multienzyme Protein Rate limiting enzyme for fat
synthesis, Contain ◊ biotin, biotin carboxylase, biotin carboxyl carrier protein,
Transcarboxylase and a regulatory Allosteric site.

• Carbamoyl phosphate synthatase ◊ rate limiting enzyme (mitochondrial) in ure


synthesis

(Q.4 Which of the following biochemical changes is not indicative of Hyperlipidemia type
5) IIa?

(a) Triglycerides normal

(b) Total cholesterol increased

(c) LDL increased

(d) HDL increased

Your Response :
Correct Answer : D
Exp (HDL increased)
:
Familial hyper cholesterolemia (IIa) ◊ one of the hyperlipidemia ◊ due to
defective LDL Receptors – C/B◊

LDL↑↑, plasma triglycerides normal,

HDL ◊ Normal or Reduced

VLDL ◊ Tends to Increase.


(Q.46) Urea cycle occurs in ?

(a) Cytosol only

(b) Mitochondria only

(c) Cytosol and Mitochondria

(d) Cytosol and endoplasmic reticulum.

Your Response :
Correct Answer : C

Exp: Cytosol and Mitochondria

METABOLIC PATHWAY SITE OF OCCURRENCE

Glycolysis Cytosol

Krebs/TCA/Citric acid cycle Mitochondrial matrix

Gluconeogenesis Partly in cytosol and in mitochondr

Glycogenolysis and Glycogenesis Cytosol

HMP shunt Cytosol

Protein synthesis Cytosol

Urea cycle Partly in cytosol and in mitochondr


Triacylglycerol biosynthesis Endoplasmic reticulum(some are
biosynthesized in mitochondria)

Ether phospholipids, Plasmalogens and PAFs Peroxisomes


biosynthesis

Sphingolipid biosynthesis Endoplasmic reticulum

Sphingomyelin biosynthesis Golgi apparatus > Plasma membran

Glycosphingolipid (Cerebrosides and Golgi apparatus


Gangliosides) biosynthesis

Cholesterol biosynthesis Endoplasmic reticulum > Cytosol

Fatty acid biosynthesis Cytosol

Monounsaturated fatty acid biosynthesis Endoplasmic reticulum

Ceramide biosynthesis from serine Endoplasmic reticulum

Bile acid biosynthesis Cytosol

N-Linked Glycoside biosynthesis Endoplasmic reticulum

O-Linked Glycoside biosynthesis Golgi apparatus

Glycosaminoglyacans biosynthesis Golgi apparatus

Elongation of fatty acid chains Endoplasmic reticulum

Fatty acid oxidation Mitochondria

Oxidation of very long chain Fatty acids Peroxisomes (initially until it reache
Octanyl stage)

(Q.47) Function of endonucleases

(a) Cut DNA at specific DNA sequences

(b) To point out the coding regions

(c) Enhancers

(d) To find out antibiotic resistance

Your
Response :
Correct A
Answer :
Exp: (cut DNA at specific DNA sequences)

Restriction Enzymes cut DNA chains at specific location: -


certain ENDONUCLEASES enzymes that cut

DNA at specific sequences within the molecule (as


opposed to EXONUCLEASES, which digest from ends of

DNA molecules), are a key tool in


recombinant DNA research.

(Q.48) The product of oxidation of odd chain fatty acids is

(a) Aceto acetyl CoA

(b) Malonyl CoA

(c) Propionyl CoA

(d) Fumaryl CoA.

Your Response :
Correct Answer : C

Exp: (Propionyl CoA)

• Oxidation of a fatty acid with an odd number of carbon atoms yields-Acetyl CoA
and molecule of propionyl

CoA ◊ oxidized by -oxidation. The propionyl residue from an odd-chain fatty is the
only part of a fatty acid that is glucogenic.
(Q.49) Which of the following findings is considered a minor criteria of CHF using the Framingham criteria?

(a) Neck vein distension

(b) Rales

(c) Positive hepatojugular reflux

(d) Extremity edema

Your Response :
Correct Answer : D

Exp: Extremity edema

Harrison’s 17th Edition, Page no 1447-1448


Use of the Framingham criteria (eight major and seven minor) is one method by
organize the signs and symptoms for the diagnosis of congestive heart fail ure. M
criteria include paroxysmal nocturnal dyspnea, neck vein dis tension, rales, cardi
acute pulmonary edema, 53 gallop, increased venous pressure, and hepatojugula
Minor criteria include extremity edema, night cough, dyspnea on exertion,
hepatomegaly, pleural effusion, vital capacity reduced by one-third from normal
tachycardia of 120 or more beats per minute. In addition, weight loss of 4.5 kg or
over 5 days of treatment may be considered as a major or minor criterion. To est
clinical diagnosis of congestive heart failure, at least one major and two minor cr
required.

(Q.50) Which is lipotropic factor

(a) HDL

(b) Insulin

(c) Choline

(d) Carnitine

Your Response :
Correct Answer : C

Exp: (choline)

Lipotropic factors: The substances that prevent the accumulation of fat in liver. E
Choline (most imp.),

methionine, betaine and -propiothetin. Also Lecithin (that contain choline), and

(Q.51) Froment' s test detects:

(a) Radial nerve palsy

(b) Median nerve palsy

(c) Ulnar nerve palsy

(d) Axillary nerve palsy


Your Response :
Correct Answer : C

Exp: Ulnar nerve palsy

JC Mahashwari, 3rd edition (Revised), Page 56

• Tests for ulnar nerve palsy:

1. Egawa's test

2. Card test

3. Froment's sign or book test - detects paralysis of adductor pollicis and first pal
interosseous muscle.

(Q.52) In prevention of retrolental fibroplasia, the oxygen should be:

(a) 10-20%

(b) 30-40%

(c) 50-60%

(d) 70-80%

Your Response :
Correct Answer : B

Exp: 30-40%

Retrolental fibroplasia or retinopathy of prematurity (ROP) is a bilateral retinal


disease seen in premature infants due to excessive oxygenation during 1st few w
life. A high concentration of oxygen leads to spasm of the retinal vessels followed
vascular dilatation and neovascularization in the periphery of retina. Progressive
neovascularization leads to vitreous hemorrhage and localized retinal detachmen
contraction of fibrovascular tissue ultimately results in total detachment of retin
lies as a white mass behind the lens, causing leukocoria and blindness. Main diffe
diagnosis is retinoblastoma in which there is usually no history of prematurity an
therapy. Oxygen concentration of 30% is considered to be safe.

(Q.53) Triradiate pelvis is seen in:

(a) Osteomalacia
(b) Senile osteoporosis

(c) Paget's disease

(d) Hypothyroidism

Your
Respons
e:
Correct A
Answer :
Exp: Osteomalacia

Triradiate pelvis is seen in


osteomalacia.

(Q.54) In a young adult patient which drug can be used that will provide only mydriasis and no cycloplegia fo
examination:

(a) Atropine ointment

(b) Phenylephrine

(c) Homatropine

(d) Tropicamide

Your Response :
Correct Answer : B

Exp: Phenylephrine

Phenylephrine is used for fundus examination as a mydriatic when cycloplegia is


required. It also reduces intraocular pressure in wide angle glaucoma.

(Q.55) Which of the following is the most common complication of extracapsular cataract surgery:

(a) Vitreous hemorrhage

(b) Opacification of posterior capsule

(c) Retinal detachment

(d) None

Your Response :
Correct Answer : B

Exp: Opacification of posterior capsule

Extracapsular extraction involves excision of a large portion of anterior capsule fo


by expression of the nucleus and cortical clean up. Advantages of this procedure
flexible loop posterior chamber lens implantation is possible and the operation is
influenced by the presence of congenital capsulo-hyaloidal adhesions so it is pref
young patients. Vitreous related complications such as vitreous loss, pupillary blo
vitreous torch syndrome and vitreous wick syndrome do not occur provided the
procedure is uneventful. Incidence of postoperative cystoid macular edema and
detachment are also less as compared with intracapsular surgery. Disadvantages
the operation is difficult to master and postoperative opacification of posterior
capsule occurs in 10-50%of cases after 3-5 years. This surgery is contraindicated
with chronic anterior uveitis.

(Q.56) Phantom tumor is:


(a) Loculated pleural effusion

(b) Sub-pulmonic effusion

(c) Fissural effusion

(d) Paracardiac effusion

Your Response :
Correct Answer : C

Exp: Fissural effusion

(Ref. Grainger & Allison's Diagnostic Radiology: A Textbook of Medical Imaging


Ed.pg. 327)

PHANTOM TUMOUR

Fissural interlobular loculation is seen particularly in heart failure and may produ
so-called phantom tumour.

Viewed in lateral view it is sharply marginated and biconvex and has a tail passin
the fissure.

The en face appearances depend on the thickness of the effusion.

A common problem in practice is the differentiation of encysted fluid in the lowe


oblique fissure from a middle lobe collapse. Observations that favour a collapsed
consolidated middle lobe rather than an effusion include non-homogeneity, a str
concave border in the lateral view, a wedge-like outline with the base reaching t
sternum, and absence of the minor fissure.

In case of doubt, a CT scan can be performed.

(Q.57) Funduscopic examination of a patient with TIA may reveal what physical finding? (See fig)

(a) Hollenhorst plaques

(b) Papilledema

(c) Cotton-wool spots

(d) A V nicking

Your Response :
Correct Answer : A

Exp: Hollenhorst plaques

On examination of the fundus, you may see Hollenhorst plaques. These are chole
emboli that are lodged in the retinal artery. They arise from an atheromatous pla
contain both cholesterol and fibrin. These emboli originate from plaques in the c
arteries, and auscultation of the carotid arteries may reveal bruits. The other fun
findings are seen in both hypertensive and diabetic retinopathy.

(Q.58) Ptosis with weakness of orbicularis oculi is seen in

(a) Polymyositis
(b) Myasthenia gravis

(c) Eaton-Lambert myasthenic syndrome

(d) Thyrotoxicosis

Your Response :
Correct Answer : B

Exp: [Myasthenia-gravis]

Common ophthalmic features of M. gravis

(a) Common - Ptosis, and diplopia, inability to maintain up gaze Cogan, 'lid-twitch
weakness of orbicularis oculi, with compromised lid-closure

(b) Uncommon - Pseudo-internuclear ophthalmoplegia

Positive ice test - The degree of ptosis improves after an ice pack is placed on the
for 2 minutes, the test is negative in non-myasthenic ptosis

(Q.59) Weakness of the extraocular muscles is seen in all the following EXCEPT

(a) Fischer syndrome

(b) Myasthenia gravis

(c) Eaton-Lambert syndrome

(d) None of the above

Your Response :
Correct Answer : D

Exp: Fisher-syndrome - on ocular varient of Gullian Barre syndrome can produce


ophthalmoplegia, with areflexia and ataxia.

Eaton-lambert myasthenic syndrome can also produce ophthalmoplegia

Giant cell (temporal) arteritis occasionally manifests as diplopia from ischemic pa


extraocular muscle.

Thyrotoxicosis - ophthalmic features

(a) Common -lid retraction, chemosis, proptosis

(b) Uncommon - superior limbic keratoconjunctivitis, keratoconjunctivitis sicca a


diplopia

(c) Rare - optic neuropathy and choroidal folds.

(Q.60) In unilateral afferent pupillary defect, when light is moved from the normal to the affected eye, there

(a) . Dilatation on the affected side and constriction in the normal eye

(b) . Dilatation in the normal eye and constriction in the affected side

(c) . Dilatation on both pupils

(d) . Constriction in both pupils

Your Response :
Correct Answer : C

Exp: [Dilatation of both pupils]

Afferent pupillary defect

I. Absolute afferent pupillary defect (Amaurotic pupil) is caused by a complete op


nerve lesion and is characterized by the following:

The involved age is completely blind (Le. no light perception)

Both pupils are equal in size

When the affected eye is stimulated by light neither pupil reacts but when the no
eye is stimulated both pupils react normally.

The near reflex is normal in both eyes.

II. Relative afferent pupillary defect. A relative pupillary defect (Marcus Gunn pup
caused by an incomplete optic nerve lesions or severe retinal disease, but never
dense cataract.

The clinical picture are those of an amaurotic pupil but more subtle. Thus the pu
respond weakly to stimulation of the diseases eye and briskly to that of the norm

The difference between the pupillary reactions of the two eyes is highlighted by
'swinging-flash light test' in which a light source is alternatively switched from on
the other and back, thus stimulating each eye in rapid succession.

First the normal eye is stimulated, resulting in brisk constriction of both pupils. T
when the light is swung to the diseases eye*, both pupils dilate instead of constr
This paradoxical dilatation of the pupils in response to light occurs because the d
produced by withdrawing the light from the normal eye outweighs the constricti
produced by stimulating the abnormal eye.

**In afferent (sensory) lesions the pupils are equal in size. Anisocoria (inequality
pupillary size) implies disease of the efferent (motor) nerve, iris or muscles of the

(Q.61) Bilateral optic disc oedema with normal CT scan is caused by

(a) Benign intracranial hypertension

(b) Malignant hypertension

(c) Brain tumours

(d) None of the above

Your Response :
Correct Answer : A

Exp: [Benign intracranial hypertension]

Pseudo tumor cerebri (Benign Intracranial Hypertension)

Symptoms - headache, diplopia, and visual disturbances due to papilledema and


abducens nerve dysfunction

Examination - reveals the papilledema and some enlargement of the blind spots
patients otherwise look well

Investigation - reveals no evidence of space occupying lesions, and the CT -scan


small or normal ventricles

Lumbar puncture confirms the presence of intracranial hypertension, but the CSF
normal

*Untreated pseudotumor cerebri leads to secondary optic atrophy and permane


loss

(Q.62) Examination of a patients visual field reveals complete blindness in left eye. Ophthalmoscopic examin
normal. At what level is lesion?

(a) . Between the optic chiasma and the lateral geniculate body

(b) . Between the retina and the optic chiasma

(c) . Between the lateral geniculate body and the visual cortex
(d) . At the medial longitudinal fasciculus

Your Response :
Correct Answer : B

Exp: Between the retina and the optic chiasma

When defects are detected in only one eye, the lesion is anterior to the optic chi
Lesions at the optic chiasma produce bitemporal hemianopia because this is whe
retinal fibers decussate. The mediallongitudinal fasciculus is involved with extrao
muscle coordination. Lesions between the geniculate body and visual cortex wou
produce a contralateral upper homonymous quadrantanopia. Because fibers sub
similar areas of the retinas become very close as they travel posteriorly to the oc
lobes (visual cortex), a lesion in the visual cortex would produce similar field defe
each eye.

(Q.63) Which of the following drugs acts via the T-type calcium channel antagonism:

(a) Ethosuximide

(b) Phenytoin

(c) Gabapentin

(d) Lamotrigine

Your Response :
Correct Answer : A

Exp: Ethosuximide

KDT, 6th Edition, Page no 407

The specific effect of ethosuximide against absence seizures appears to be due to


ability to block T-type calcium channels at concentrations that do not affect othe
channels.

(Q.64) Bosentan is a new drug being used for:


(a) PAH
(b) RA
(c) DM
(d) CML
Your Response :
Correct Answer A
:
Exp: PAH
Harrison’s 17th Edition, Page no 1578
It is endothelin receptor antagonist. It is a vasodilator now being
used for PAH & Raynaud’s disease.
(Q.65) Which of the following organisms is a natural transformer?

(a) Escherichia coli

(b) Neisseria gonorrhoeae

(c) Plasmodium vivax

(d) Pseudomonas aeruginosa

Your Response :
Correct Answer : B

Exp: Neisseria gonorrhoeae

Transformation was first described in S. pneumoniae by Griffith. Other bacteria w


naturally transform include N. gonorrhoeae, Hemophilus spp. and H. pylori

(Q.66) Which of the following structures is found only in Gram-negative microorganisms?

(a) Cell envelope

(b) Exotoxin

(c) Peptidoglycan

(d) Periplasmic space

Your Response :
Correct Answer : D

Exp: Periplasmic space

Structures exclusively found in gram-negative bacteria include: periplasmic space


membrane, LPS and pili or fimbriae; while the structure exclusively found in gram
bacteria is teichoic acid.

(Q.67) Necrobiosis lipoidica diabeticorum is more marked on:


(a) Back of legs

(b) Front of legs

(c) Face

(d) Trunk

Your Response :
Correct Answer : B

Exp: Front of legs

Harrison’s 17th Edition, Page no 2293


Necrobiosis lipoidica diabeticorum is a plaque-like lesion with a central yellowish
surrounded by a brownish border & is usually found over the anterior surfaces o

(Q.68) Growing phase of hair is:

(a) Catagen and telogen

(b) Telogen

(c) Anagen

(d) Catagen

Your Response :
Correct Answer : C

Exp: Anagen

Telogen is the resting stage of the hair growth cycle.

Anagen is the growth stage of hair development.

Catagen is the intermediate phase of the hair-growth cycle, between the growth
anagen stage and the resting or telogen phase.

(Q.69) External branch of Superior Laryngeal nerve supply

(a) Cricothyroid

(b) Thyroarytenoid

(c) Post cricoarytenoid


(d) Lateral Cricoarytenoid

Your Response :
Correct Answer : A

Exp: Cricothyroid

Superior Laryngeal nerve divides within the carotid sheath at about the level of t
bone into two branches, the external and internal laryngeal nerves.

Internal laryngeal nerve is a branch of the vagus nerve. It descends within the ca
sheath posterior to the internal carotid artery and then passes anteromedially at
level of thyrohyoid membrane. It pierces the thyrohyoid membrane to emerge w
laryngeal part of the pharynx in the piriform recess. From the piriform recess, the
fibres distribute as, according to their type:

Somatic sensory fibres from the mucosae of:

Larynx above the vocal cords

Epiglottis

Valleculae

Special visceral sensory(SVA) fibres that carry the sensation of taste from the reg
the posterior most tongue/ valleculae

The somatic sensory fibres have their cell bodies in the inferior vagal ganglion an
synapse with the sensory nucleus of the trigeminal nerve within the medulla.

The special visceral sensory fibres have their cell bodies in the inferior vagal gang
synapse in the nucleus of tractus solitarius(NTS)

External laryngeal nerve passes inferiorly within the carotid sheath, posterior to
common carotid artery, and then anteromedially next to the superior thyroid art

It ramifies on the surface of the inferior constrictor muscles before piercing them
supplies special visceral motor (SVE) fibres to the cricothyroid muscle. Also, it ma
a few motor fibres to the cricopharyngeus part of the inferior constrictors as it pa
through. Both sets of nerve fibres originate from nuclei within the nucleus ambig
the medulla.

(Q.70) Most common cause of Athlete's foot is:

(a) Histoplasma capsulatum


(b) Trichophyton rubrum

(c) Pityrosporum furfu

(d) Candida albicans

Your
Response :
Correct B
Answer :
Exp: Trichophyton rubrum

Harrison’s 17th Edition, Page no 751


Most common cause of athlete's foot is Trichophyton
rubrum.

(Q.71) Which change is not found in a dead born child?


(a) Rigor mortis at birth
(b) Mummification
(c) Adipocere formation
(d) Maceration
Your Response :
Correct Answer : C
Exp: Adipocere formation
A dead-born child is one which has died in uterus, and may show signs of rigor m
maceration and mummification, i.e. dried up and shrivelled.

Rigor mortis-Most common cause is antenatal haemorrhage in mother.

Maceration-Foetus surrounded by liquor amnii but with exclusion of air, macerat


commence immediately and characterized by softening of dead fetus and degen
of tissue.

(Q.72) Which of the following is shortest acting mydriatic and


cycloplegic drug?

(a) Scopolamine

(b) Homatropine

(c) Cyclopentolate

(d) Tropicamide

Your
Response :
Correct D
Answer :
Exp: Tropicamide

KDT, 6th Edition, Page no 111

Mydriatric drugs Onset of action/duration


of action

Atropine 30-40 min/l-3 hours

Homatropine 45-60 min/1-3 days

Cyclopentolate 30-60 min/24 hours

Scopolamine 40-60 min/l 0-12 hours

Tropicamide 20-40 min/3-6 hours

(Quickest and briefest)

Tropicamide is a quickest and briefest in action,


used as short acting mydriatic for funduscopy.

(Q.73) The intra-abdominal pressure during laparoscopy should be set between:

(a) 5- 8 mm of Hg

(b) 10 - 15 mm of Hg

(c) 2 - 25 mm of Hg

(d) 30 - 35 mm of Hg

Your Response :
Correct Answer : B

Exp: 10 - 15 mm of Hg

For laparoscopic surgery, pneumoperitoneum is created by inflating the abdome


about 1-4L of gas (CO2), but for diagnostic purposes nitrous oxide or room air or
can be used. Volume of gas varies from 1-4L depending upon the patient, but in
intra-abdominal pressure should not exceed 20 mmHg. Flow rate of gas is about
with a pressure not exceeding 20 mmHg.

(Q.74) The following statements are true about intrauterine devices (IUD)
except:

. Levonorgestrel releasing IUD has an effective life of 5


(a) years

. IUD can be used for emergency contraception within


(b) 5 days

(c) . Pain and bleeding are frequent complications

(d) . Multiload Cu-375 is a third generation IUD

Your
Response :
Correct D
Answer :
Exp: Multiload Cu-375 is a third generation IUD

Generation Device Example

First Non-medicated Lippes Loop


generation or inert IUDs

Second Copper IUDs Cu-7, Cu T – 200;


generation T Cu-220 C, T Cu-
380 A or

Ag; Nova T;
Multiload devices
(ML - Cu -

250 & ML- Cu -


375)

Third Hormone Progestasert


generation releasing IUDs

(Q.75) Drugs aggravating myasthenic symptoms are all the following except:

(a) Erythromycin

(b) Propranolol

(c) Ciprofloxacin

(d) Tetracyclines
Your Response :
Correct Answer : D

Exp: Tetracyclines

Harrison’s, 17th Edition, Page no 2677, Table 381-4

Table Drugs with Interactions in Myasthenia Gravis (MG)

Drugs That May Exacerbate MG

Antibiotics

Aminoglycosides: e.g., streptomycin, tobramycin, kanamycin

Quinolones: e.g., ciprofloxacin, levofloxacin, ofloxacin, gatifloxacin

Macrolides: e.g., erythromycin, azithromycin, telithromycin

Nondepolarizing muscle relaxants for surgery

D-Tubocurarine (curare), pancuronium, vecuronium, atracurium

Beta-blocking agents

Propranalol, atenolol, metoprolol

Local anesthetics and related agents

Procaine, xylocaine in large amounts

Procainamide (for arrhythmias)

Botulinum toxin

Botox exacerbates weakness

Quinine derivatives

Quinine, quinidine, chloroquine, mefloquine (Lariam)

Magnesium

Decreases ACh release

Penicillamine
May cause MG

Drugs with Important Interactions in MG

Cyclosporine

Broad range of drug interactions, which may raise or lower cyclosporine levels.

Azathioprine

Avoid allopurinol—combination may result in myelosuppression.

(Q.76) An ectopic pregnancy is shed as

(a) Decidua vera

(b) Decidua basalis

(c) Decidua capsularis

(d) Decidua rubra

Your Response :
Correct Answer : A

Exp: Decidua vera

The endometrial lining of the uterus is called decidua during pregnancy, If there i
fertilization and pregnancy occurs, the decidua differentiated into

Decidua basalis - becomes the maternal portion of the placenta

Decidua - capsularis - the thin layer covering the ovum

*Decidua vera or parietalis - which is the rest of the decidua lining the uterine ca
outside the site of implantation

*The decidua develops all the characteristic of intra uterine pregnancy except th
contains no evidence of chorionic villl

*When the ovum is dead it is either disintegrated and comes out piecemeal or co
single piece (decidual cast)

Functions of decidua
Provide the nidus for implantation of the fertilized ovum
Provide nutrition (glycogen, fat) to the growing ovum

The growing substance of the decidua consisting of mucopolysaccharides limits d


penetration by the trophoblastic cells, a protective action

Provide the basal plate of the placenta.

(Q.77) Clomiphene citrate is indicated in

(a) Stein-Leventhal syndrome

(b) Asherman’s syndrome

(c) Carcinoma endometrium

(d) All of the above

Your Response :
Correct Answer : A

Exp: Stein-Leventhal syndrome

If the patient wishes to become pregnant, clomiphene or other drugs can be emp
for induction of ovulation for female infertility
Clomiphene - can sometimes stimulate a man's own pituitary gonadotropins (wh
pituitary is intact) there by increasing testosterone and sperm production for ma
infertility

Clomiphene citrate is non steroidal estrogen antagonist it induces ouulation in 60


women with stein-Ieventhal syndrome (PCOS) and is the initial treatment of choi
Adverse effects - Poly cystic ovaries, multiple pregnancy hot flushes, gastric upse
vertigo, allergic dermatitis, risk of ovarian tumor may be increased

(Q.78) Endometrial hyperplasia is seen in

(a) Endodermal sinus tumour

(b) Dysgerminoma

(c) Polycystic ovarian disease

(d) Carcinoma of cervix

Your Response :
Correct Answer : C
Exp: Polycystic ovarian disease

Polycystic ovarian disease - endometrial hyperplasia due to high content of estro


largely estrone, by extraglandular aromatization of circulating androstenedione.

Endometrial hyperplasia is common in carcinoma carcinoma endometrium

Treatment - simple endometrial hyperplasia calls for cyclic progestin therapy


(medroxyprogesterone or norethindrone)

If endometrial hyperplasia with atypical cells or carcinoma of the endometrium i


hysterectomy is necessary

(Q.79) All the following are indications for termination of pregnancy in an APH patient EXCEPT

(a) 37 weeks

(b) IUD

(c) Transverse lie

(d) Significant bleeding

Your Response :
Correct Answer : C

Exp: . The definite management - comprises prompt delivery, this is considered when

The patient has her first bout of bleeding after 37 completed weeks . Successful
conservative treatment brings the patients upto 37 weeks . If the initial or a subs
bout of bleeding is very severe

Patient is in labour

Evidence of maternal or fetal jeopardy

Intrauterine fetal death

(Q.80) All the following are TRUE about HCG EXCEPT

(a) Has luteotrophic action

(b) Acts on the same receptors as LH

(c) Secreted by cytotrophoblast


(d) It is a glycoprotein

Your Response :
Correct Answer : C

Exp: Secreted by cytotrophoblast

Human chorionic gonadotrophin (hCG) is a glycoprotein


It consists of a hormone non specific a subunit is biochemically similar to LH, FSH
TSH

Hormone specific ~ subunit is relatively unique to hCG.

hCG is produced by syncytiotrophoblast of the placenta and secreted into the blo
both mother and fetus

(Q.81) A 30-year-old woman presents with amenorrhoe(A.) of 6 weeks' duration and a lump in the right iliac
foss(A.)Investigation of choice is

(a) Ultrasound abdomen

(b) Laparoscopy

(c) Shielded CT scan

(d) X-ray

Your Response :
Correct Answer : A

Exp: Ultrasound abdomen

Ultrasound - simple, noninvasive, painless diagnostic procedure that has the ad


of being of any radiation hazard It is specially helpful in a child, virgin or an obe
woman
Generally this scan is done when patients bladder is full as it helps to elevate the
out of the pelvis, and displaces the gas filled bowel loops away, thus providing th
sonologist with a window to image the pelvic organs.

(Q.82) Paramesonephric duct develop into:

(a) Vas deferens

(b) Seminal vesicle


(c) Ureter

(d) Uterus

Your Response :
Correct Answer : D
Exp: Uterus

Male Female

Mesonephric duct Duct of epididymis Duct of epoophoron

(Wolffian duct) Ductus deferens Part of bladder and


urethra

Ejaculatory duct

Part of bladder and prostatic


urethra

Female Appendix of testis Uterine tube

(Paramesonephric or Prostatic utricle Uterus


Mullerian duct)

Vagina (?)

(Q.83) In the male, the homologue of the vaginal artery is which of the following?

(a) Obturator artery

(b) Internal pudendal artery

(c) Middle rectal artery

(d) Inferior vesical artery


Your Response :
Correct Answer : D
Exp: Inferior vesical artery
All of the listed choices are branches of the internal iliac artery The inferior vesic
in the male supplies the seminal vesicle, prostate, fundus of the bladder, distal u
and the vas deferens. In the female, the vaginal artery supplies the vagina, urinar
bladder, and pelvic portion of the urethra. The obturator artery gives off muscula
nutrient branches within the pelvis and then leaves the pelvis via the obturator c
supply the thigh. The internal pudendal artery crosses the piriformis muscle, exit
pelvic cavity via the greater sciatic foramen, and enters the ischiorectal fossa via
lesser sciatic foramen. It supplies the external genitalia (penis and clitoris). The m
rectal artery supplies the inferior rectum and forms important anastomoses with
rectal arteries. The umbilical artery gives off the superior vesical artery in both se
distal portion degenerates to form the medial umbilical ligament.

(Q.84) False regarding tympanic membrane is:

(a) It has sensory supply via auriculotemporal nerve

(b) It is inclined at an angle of 35° to the meatus

(c) Lined by stratified epithelium in continuity with external auditory canal

(d) Tympanic membrane is attached to annulus ring made of fibrous cartilage

Your Response :
Correct Answer : B

Exp: It is inclined at an angle of 35° to the meatus

Tympanic membrane or eardrum or drumhead or myringa is a thin, translucent p


wall between the external~ acoustic meatus and the middle ear. It is oval in shap
measures 9 x 10 mm and is placed obliquely at 55° ' with the floor of the meatus
downwards, forwards and laterally. The rim of thetympanic membrane: consists
fibrocartilage ring which is deficient in its superior part. This ring fits in a bony su
(tympanic' annulus) which lies at the medial end of the external auditory meatus
notch of Rivinus is a superior deficiency of both the cartilaginous annulus and the
annulus which lies medial to the pars flaccida of the drum. Outer layer of the tym
membrane is made up of the squamous ephithymum Which is continuous with t
the meatus; middle layer by fibrous tissue; and inner layer by mucus membrane
continuous with the lining of tympanic cavity. Anteroinferior part of outer surfac
tympanic membrane is supplied by auriculotemporal nerve while the posterosup
supplied by the auricular branch of vagus nerve. Inner surface is supplied by tym
branch of glossopharyngeal nerve through the tympanic plexus.

(Q.85) Not seen in fracture of maxilla:

(a) Anesthesia of upper lip

(b) CSF rhinorrhea

(c) Mobility of the maxilla


(d) Malocclusion

Your Response :
Correct Answer : A

Exp: Anesthesia of upper lip

Clinical Features of Fracture of Maxilla


Elongation of the mid-face;

Malocclusion of teeth with anterior open bite;

Mobility of the maxilla;

CSF rhinorrhea due to injury to cribriform plate in Le Fort II and III fractures.

(Q.86) In which of the following conditions tonsillectomy is indicated:

(a) Physiological enlargement

(b) Sleep apnea

(c) Aphthous ulcer in pharynx

(d) Acute tonsillitis

Your Response :
Correct Answer : B

Exp: Sleep apnea

Indications of Tonsillectomy

Absolute indications Relative indications

Recurrent attacks of tonsillitis (most Diphtheria carriers


common)

Upper airway obstruction Systemic disease due to p-hemolytic


streptococcus

(Rheumatic fever, nephritis)

Cor pulmonale Halitosis (bad smell from mouth due to pus i


tonsils)
Peritonsillar abscess (Quinsy) Documented recurrent acute tonsillitis

Sleep apnea Chronic tonsillitis

Suspicion of malignancy Tonsillitis resulting in febrile convulsions

(Q.87) All are false regarding infraglottic carcinoma larynx except

(a) Spreads to submental nodes

(b) It is the most common laryngeal carcinoma

(c) Commonly spread to mediastinal nodes

(d) Second most common laryngeal carcinoma

Your Response :
Correct Answer : C

Exp: Commonly spread to mediastinal nodes

Subglottic or infraglottic tumor are mainly squamous cell type and are more com
male smokers although subglottis is the rarest site of laryngeal cancers. Subglott
small area which extends from the lower border of cricoid to the under surface o
cord. They spread to the thyroid gland in 20% of cases, involve the strap muscles
in 20% of cases, and give rise to cervical lymph node metastasis in 20% of cases.
para tracheal lymph nodes present in mediastinum may get involved.

(Q.88) In infants the bilateral choanal atresia generally presents as:

(a) Difficulty in feeding

(b) no complaints

(c) Dysphagia

(d) Difficulty in breathing

Your Response :
Correct Answer : D

Exp: Difficulty in breathing

Choanal atresia is usually unilateral but bilateral cases do occur and present at bi
severe respiratory distress and is a neonatal emergency. Emergency treatment is
insertion of an oral airway which is fixed in place to ensure respiration until cura
surgery can be undertaken.
(Q.89) One of the following is not a common ENT manifestation of AIDS

(a) Recurrent Aphthous Ulcer

(b) Hairy Leukoplakia of Tongue

(c) Thrush

(d) Viral labyrinthitis

Your Response :
Correct Answer : D

Exp: Viral labyrinthitis

Oropharyngeal and gastrointestinal diseases are common features of HIV infectio


are most frequently due to secondary infections. In addition, oral and gastrointe
lesions may occur with Kaposi’s sarcoma and lymphoma. Oral lesions, including
hairy leukoplakia, and aphthous ulcers, are particularly common I patients with u
HIV infection. Thrush, due to candida infection, and oral hairy leukoplakia, presu
to EBV, are usually indicative of fairly advanced immunologic decline; they gener
occur in patients with CD4+ T cell counts of <300/L.

(Q.90) All of the following are true about radical mastoidectomy except:

(a) Bone and muscles from the middle ear cavity are removed

(b) Facial bridge is removed

(c) Esutachian tube is curetted and plugged

(d) There is no indication of this surgery these days

Your Response :
Correct Answer : D

Exp: There is no indication of this surgery these days

Radical mastoidectomy is indicated in carcinoma middle ear, carcinoma external


skill base osteomyelitis

(Q.91) Pulsatile tumor of external auditory meatus is:


(a) Malignancy of middle ear

(b) Polyp

(c) Glomus tumor

(d) Hemangioma

Your Response :
Correct Answer : C

Exp: Glomus tumor

Pulsation sign is positive in glomus tumor i.e. when the ear canal pressure is raise
Siegle's speculum the tumor pulsates vigorously and blanches.

(Q.92) Undisplaced fracture of zygoma is managed by:

(a) Closed reduction

(b) Open reduction

(c) No specific treatment

(d) Wiring to frontal bone

Your Response :
Correct Answer : C

Exp: . No specific treatment ygoma is the second most common bone to be fractured
the nasal bone. Direct trauma is the usual cause. Treatment is required only for t
displaced fractures. Open reduction and internal wire fixation is the best option.

(Q.93) Alcohol abuse is associated with an increased risk of all the following types of cancer EXCEPT:
(a) Breast
(b) Esophageal
(c) Liver
(d) Cervical
Your Response :
Correct Answer : D
Exp: Cervical
Harrison’s 17th Edition, Page no 2726
Alcohol abuse increases the risk of esophageal and laryngeal cancer, as well as liv
cancer. A modest increase in the risk of breast cancer has been demonstrated in
who drink more than 2 drinks per day. Cancer of the cervix has not been associat
alcohol use.

(Q.94) The angiogram depicted below is most typical of the patient whose history includes (See fig)

(a) Cigarette smoking

(b) Alcoholism

(c) Hypertension

(d) Diabetes

Your Response :
Correct Answer : A

Exp: Cigarette smoking

(Schwartz, 8/e, pp 957–964.) The angiogram presented in the question demonstr


isolated segment of atherosclerotic occlusion of the superficial femoral artery. Pa
who have isolated femoropopliteal disease tend to be smokers, whereas those w
isolated tibioperoneal disease frequently are diabetic. Hypertension and hyperlip
predispose to accelerated atherosclerosis. On the other hand, type I
hyperlipoproteinemia (hyperchylomicronemia), which is associated with dramati
of plasma triglyceride and formation of xanthomas, does not cause accelerated v
disease.

(Q.95) Liposarcoma is a soft tissue sarcoma which cause significant morbidity. Extrapulmonary metastases a
common in which of the following subtypes of liposarcoma

(a) Pleomorphic
(b) Myxoid

(c) Lipoid

(d) Anaplastic

Your Response :
Correct Answer : B

Exp: Myxoid

Liposarcoma is a very histologically heterogenous soft tissue sarcoma. The myxoi


subtype is the most common. Those patients with myxoid subtype are more likel
have a extrapulmonary metastases than those with pleomorphic tumors. This su
that patient with myxoid liposarcomas should undergo abdominal, retroperitone
extrapleural chest imaging during initial staging and posttreatment followup.

(Q.96) The murexide test is used to detect the presence of

(a) Pyrophosphate

(b) Triglyceride

(c) Uric acid

(d) Creatinine phosphokinase

Your Response :
Correct Answer : C

Exp: Uric acid

The murexide test is used for testing for the presence of uric acid. A few drops o
acid are added to the suspected substance. The mixture is evaporated to dryness
moistened with ammonium hydroxide if uric acid is present a purple colour resul

(Q.97) The gait cycle consists of the following parts except

(a) Heel strike

(b) Stance phase

(c) Toe off

(d) Atonic phase


Your Response :
Correct Answer : D

Exp: Atonic phase

Though different authors have divided the gait cycle (the sequence of events in e
step) basically1 the gait cycle consists of four parts: Heel Strike, stance phase, toe
Swing phase. A limp is mainly an abnormal gait.

(Q.98) All the following rare connective tissues disorders have one thing in common, generalized hyper mob
severe laxity at the joints except

(a) Marfan's syndrome

(b) Ehlers Danlos syndrome

(c) Boerhaave's disease

(d) Larsen's disease

Your Response :
Correct Answer : C

Exp: Boerhaave's disease

Boerhaave's disease is oesophageal perforation. It is common after upper GI end


Patient has severe pain chest. X-ray chest shows widening of mediastinum. Treat
urgent surgery.

(Q.99) In Carpal tunnel syndrome the Durkans test is used for assessment of

(a) Paraesthesia in response to compression.

(b) Paraesthesia in response to position.

(c) Perception of patient position

(d) Stiffness of fingers

Your Response :
Correct Answer : A

Exp: Paraesthesia in response to compression.

Tests for Nerve Compression in Carpal tunnel syndrome (CTS)


Test How Performed Condition Positive Result Interpre
Tested of Positi
Result

Phalen test Elbows on table, Paresthesia in Numbness or Probable


forearms vertical, response to tingling on (sensitiv
wrists flexed position radial digits specifici
within 60 s

Percussion test Lightly tap along Site of nerve “Electric” Probable


(Tinel sign) median nerve from lesion tingling positive
proximal to distal response in wrist (se
fingers 0.60, sp
0.67)

Carpal tunnel Direct compression Paresthesia in Paresthesia Probable


compression test of median nerve at response to within 30s (sensitiv
(Durkan) carpal tunnel compression specifici

Hand diagram Patient marks site of Patient's Markings on Probable


pain or altered perception of palmar side of (sensitiv
sensation on symptoms radial digits, specifici
outlined hand without negative
diagram markings in predictiv
palm 0.91)

Hand volume Hand volume Hand volume Hand volume Probable


stress test measured by increased by dynamic
displacement, repeat ≥10 mL
after 7-min stress
test and 10-min rest

Direct Wick or infusion Hydrostatic Resting Hydrost


measurement of catheter placed in pressure in pressure ≥25 compres
carpal tunnel carpal tunnel resting and mm Hg believed
pressure provocative (variable and probabl
positioning technique of CTS
related)

Static 2-point Determine minimal Innervation Failure to Advance


discrimination separation of two density of determine dysfunct
distinct points when slow-adapting separation of at
applied to palmar fibers least 5 mm
finger tip
Moving 2-point As above, with Innervation Failure to Advance
discrimination movement of the density of determine dysfunct
points fast-adapting separation at
fibers least 4 mm

Vibrometry Vibrometer placed Threshold of Asymmetry Probable


on palmar side of fast-adapting compared with (sensitiv
digit, amplitude at fibers contralateral
120 Hz, increased to hand or median
threshold of to ulnar in
perception, compare ipsilateral hand
median and ulnar
bilaterally

Semmes- Monofilaments of Threshold of Value >2.83 Median


Weinstein increasing diameter slowly impairm
monofilaments touched to palmar adapting (sensitiv
side of digit until fibers
patient can
determine which
digit is touched

Distal sensory Orthodromic Latency, Latency >3.5 ms Probable


latency and stimulus and conduction of or asymmetry
conduction recording across sensory fibers of conduction
velocity wrist velocity >0.5
m/s versus
opposite hand

Distal motor Orthodromic Latency, Latency >4.5 ms Probable


latency and stimulus and conduction or asymmetry
conduction recording across velocity of of conduction
velocity wrist motor fibers velocity >1 m/s
of median
nerve

Electromyography Needle electrodes Denervation Fibrillation Advance


placed in muscle of thenar potentials, median
muscles sharp waves, compres
increased
insertional
activity

.
(Q.100) Brisk percussion along the course of an injured nerve may elicit a tingling sensation in the distal distr
of the nerve. This is the:

(a) Tinel's sign

(b) Adson's test

(c) Babinski's sign

(d) Faber's sign

Your Response :
Correct Answer : A

Exp: Tinel's sign

Carpal tunnel syndrome is another common disorder of the upper extremity an


from compression of the median nerve within the carpal tunnel. Manifestations
paresthesia in the thumb, second, third, and radial half of the fourth finger and,
times, atrophy of thenar musculature. Carpal tunnel syndrome is commonly ass
with pregnancy, edema, trauma, osteoarthritis, inflammatory arthritis, and infilt
disorder (e.g., amyloidosis). With each test, paresthesia in a median nerve distri
induced or increased by either “thumping” the volar aspect of the wrist (Tinel’s
pressing the extensor surfaces of both flexed wrists against each other (Phalen’

(Q.101) All the following are instances where ultrasound helps in diagnosis in orthopaedics except

Detecting intraarticular fluid, hence useful in diagnosis of a synovial effusion


(a) monitor progress of an irritable hip.

(b) Diagnosis of rotator cuff tears.

(c) Screening of newborn babies for congenital dislocation of hip

(d) Progress of healing following fracture.

Your Response :
Correct Answer : D

Exp: Progress of healing following fracture

A synovial effusion is well identified by ultrasound. The cartilaginous femoral he


acetabular labrum can be clearly identified and their relationship with each othe
whether the hip is normal or abnormal. Progress of fracture healing is assessed
clinically and by X- ray.
(Q.102) Which of the following vertebral fractures is stable

(a) Wedge compression

(b) Chance fracture

(c) Burst fracture

(d) Fracture dislocation

Your Response :
Correct Answer : A

Exp: Wedge compression

Denis has divided the vertebral column into three structured columns anterior,
and posterior. Fracture involving one column is stable and involving two or thre
unstable. Wedge compression, fracture involves only the anterior column and h
stable. Burst fracture involves anterior and middle columns and the others invo
the three columns.

(Q.103) All of the following come under immediate response to Spinal cord injury (within minutes) except ?

(a) Petechial hemorrhages

(b) Hemorrhage in central gray matter

(c) Expansion of postcapillary venules

(d) Radial expansion of hemorrhage

Your Response :
Correct Answer : D

Exp: Radial expansion of hemorrhage

REF Rockwood & Green's Fractures in Adults, 6th Edition Chapter 37

Biologic Response to Spinal Cord Injury

Time from Biologic Changes at Cord Injury Site


Injury

First few Hemorrhage in the central gray matter and


minutes anterior hornsPetechial hemorrhagesDistended
postcapillary venulesRed blood cells in
perivascular spaces

1 hour Endothelial cell disruptionSeparation of


endothelial junctionsVacuolation and swelling of
endothelial cellsCraters in capillary endothelium

1 to 6 hours Necrotic changes in gray matterCytoplasmic


eosinophilia in gray matter neuronsGhost
cellsShrunken neurons, loss of Nissl bodies,
irregular shapeEosinophilic changes in
perikaryaApoptosis

4 to 8 hours Radial expansion of hemorrhageHemorrhage in


lateral columnsAneurysmal dilatation and
rupture of arteriolesMicrothrombi in
capillariesGranulated plateletsNecrotic changes
in white matterGranular appearance,
swellingSeparation of axon from its myelin
sheathAccumulation of organelles in
axonsRetraction bulbs

4 hours to 1 Edema formationVasogenic edemaFiltration


week edema

6 hours to 1 Inflammatory cell infiltrationNeutrophil


week infiltrationMonocyte infiltration

2 days to 2 CNS reactivityActivation of microgliaIncreased


weeks number of processes (pseudopods)Up-regulation
of surface antigensPhagocytic vacuoles

1 week to 4 Activation of
weeks astrocytesHypertrophyProliferationAccumulation
at margins of lesionApoptosis in white matter

After 2 Cavity and scar formation


weeks (gliosis)DemyelinationInflammatory
angiogenesisFormation of network of astrocyte
processesWallerian degeneration

CNS, central nervous system.

(Q.104) HMB-45 is a IHC stain used for diagnosis of


(a) Melanoma

(b) Lymphoma

(c) Ca Bladder

(d) Ca Larynx

Your
Response :
Correct A
Answer :
Exp: Melanoma

Immunohistochemical Stains Useful in detecting neoplasms

Tissue Marker Diagnosis

1 Estrogen and progesterone receptors Breast cancer

2 BRST-1 Breast cancer

3 Gross cystic disease fibrous protein-15 Breast cancer

4 Thyroid transcription factor 1 Lung and thyroid


cancer

5 Thyroglobulin Thyroid cancer

6 Chromogranin, synaptophysin, neuron Neuroendocrine


specific enolase cancer

7 CDX-2 Gastrointestinal
cancer

8 Calretinin, mesothelin Mesothelioma

9 Leukocyte common antigen Lymphoma

10 S-100, HMB-45 Melanoma

11 URO-III, thrombomodulin Bladder cancer

12 Α-Fetoprotein Hepatocellular
cancer,
Germ cell cancer,

Hepatoblastoma

13 β-Human chronic gonadotropin Germ cell cancer

14 Prostate specific antigen Prostate cancer

15 Cytokeratin Carcinomas

(Q.105) Complement components (C2, C4) belong to which of the following MHC group

(a) Class I

(b) Class II

(c) . Class III

(d) None of the above

Your Response :
Correct Answer : C

Exp: Class III

Class I and class II genes encode cell surface glycoproteins while class III encode
complement components.

(Q.106) Biopsy of a transplanted kidney revealed arteritis with thrombosis and presence of cortical necrosis.
features are classical of

(a) Graft-versus host disease

(b) (B.) Acute rejection


(c) Chronic rejection

(d) Hyperacute rejection

Your Response :
Correct Answer : B

Exp: Acute rejection

Acute vascular rejection is characterized by arteritis and thrombosis leading to c


necrosis. Endothelitis is seen in acute cellular rejection also.

(Q.107) Which of the following organ is not classically involved in graft-versus host disease?

(a) Skin

(b) GIT

(c) Liver

(d) Kidney

Your Response :
Correct Answer : D

Exp: Kidney

Kidney is not involved. All the other organs have high cell turn over and are invo
GVHD.

(Q.108) Which of the following decreases during aerobic exercise?

(a) Circulating blood volume

(b) Heart rate

(c) Skin temperature

(d) Cerebral blood flow

Your Response :
Correct Answer : A

Exp: Circulating blood volume

Ganong, 22nd Edition, Page no 632-635


Under no circum stances, circulating blood volume decreases during aerobic exe
The decrease in volume is the result of increased filtration of water out of the ca
within the exercising muscles. Increased filtration occurs because more capeline
the exercising muscles are filled with blood and the pressure within each capilla
higher. The increased sympathic accompanying the exercise causes an increase
rate, which in turn increases the cardiac output The increase in cardiac output c
increase in blood pressure, despite the decreased peripheral resistance. The ski
temperature increases as more blood is diverted to the skin for the purpose of
eliminating the heat generated by the exercising muscles. Brain blood flow is ke
constant by autoregulation

(Q.109) Small, bland, easily detachable valvular vegetations attached to lines of closure are characteristic of?

(a) Bacterial endocarditis

(b) Libmann Sacks endocarditis

(c) NBTE

(d) Rheumatic carditis

Your Response :
Correct Answer : C

Exp: NBTE

Infective characterized by microbial invasion of heart valves or mu


endocarditis endocardium-often with destruction of the underlying ca
tissues (valve perforation, ring abscess).
Bulky, friable vegetations composed of necrotic debris, th
and organisms.
Thromboemboism common.
Libman-Sacks SLE can be associated with Libman-Sacks endocarditis
endocarditis in which there are many flat, reddish-tan vegetations spre
over the mitral valve and chordae.
vegetations on both sides of valve
do not embolize.
NBTE (nonbacterial typically exhibits small, bland vegetations, usually attache
thrombotic line of closure.
endocarditis) Easily detachable.
NBTE in debilitated cancer and sepsis patients is marantic
endocarditis.
Rheumatic carditis Vegetations larger than 10 mm (not well penetrated by
antibiotics)
Along the line of closure
Aschoff bodies

(Q.110) Intimal fibrosis of the vessel wall is characteristic of

(a) Hyper acute rejection

(b) Acute rejection

(c) Chronic rejection

(d) GVHD

Your Response :
Correct Answer : C

Exp: Chronic rejection

Chronic rejection takes months to years after transplant. There is intimal fibrosi
resulting in renal ischemia manifested by glomerular loss, interstitial fibrosis, tu
lossand loss of renal parenchyma.

(Q.111) The most common type of lupus nephritis is

(a) Class I

(b) Mesangial lupus GN

(c) Focal proliferative GN

(d) Diffuse proliferative GN

Your Response :
Correct Answer : D

Exp: Diffuse proliferative GN

It is Class IV lupus nephritis. More than 50% of glomeruli are involved in class IV

Classification of Lupus Nephritis (International Society of Nephrology and Ren


Pathology Society)

Class I: Minimal Mesangial Lupus Nephritis

Normal glomeruli by light microscopy, but mesangial immune deposits by


immunofluorescence.

Class II: Mesangial Proliferative Lupus Nephritis

Purely mesangial hypercellularity of any degree or mesangial matrix expansion


light microscopy, with mesangial immune deposits. A few isolated subepithelia
subendothelial deposits may be visible by immunofluorescence or electron
microscopy, but not by light microscopy.

Class III: Focal Lupus Nephritis

Active or inactive focal, segmental or global endo- or extracapillary


glomerulonephritis involving <50% of all glomeruli, typically with focal subendo
immune deposits, with or without mesangial alterations.

Class III (A): Active lesions—focal proliferative lupus nephritis

Class III (A/C): Active and chronic lesions—focal proliferative and sclerosing lup
nephritis

Class III (C): Chronic inactive lesions with glomerular scars—focal sclerosing lup
nephritis

Class IV: Diffuse Lupus Nephritis

Active or inactive diffuse, segmental or global endo- or extracapillary


glomerulonephritis involving 50% of all glomeruli, typically with diffuse
subendothelial immune deposits, with or without mesangial alterations. This cl
divided into diffuse segmental (IV-S) lupus nephritis when 50% of the involved
glomeruli have segmental lesions, and diffuse global (IV-G) lupus nephritis whe
of the involved glomeruli have global lesions. Segmental is defined as a glomeru
lesion that involves less than one-half of the glomerular tuft. This class includes
with diffuse wire loop deposits but with little or no glomerular proliferation.

Class IV-S (A): Active lesions—diffuse segmental proliferative lupus nephritis

Class IV-G (A): Active lesions—diffuse global proliferative lupus nephritis

Class IV-S (A/C): Active and chronic lesions—diffuse segmental proliferative and
sclerosing lupus nephritis

Class IV-G (A/C): Active and chronic lesions—diffuse global proliferative and
sclerosing lupus nephritis

Class IV-S (C): Chronic inactive lesions with scars—diffuse segmental sclerosing
nephritis
Class IV-G (C): Chronic inactive lesions with scars—diffuse global sclerosing lupu
nephritis

Class V: Membranous Lupus Nephritis

Global or segmental subepithelial immune deposits or their morphologic seque


light microscopy and by immunofluorescence or electron microscopy, with or
without mesangial alterations. Class V lupus nephritis may occur in combination
class III or IV, in which case both will be diagnosed. Class V lupus nephritis may
advanced sclerosis.

Class VI: Advanced Sclerotic Lupus Nephritis

90% of glomeruli globally sclerosed without residual activity.

(Q.112) LE cells represent –

(a) Damageal RBCs

(b) Damaged host cell

(c) Neutrophits containing damaged cell

(d) Lymp hocyte with complement

Your Response :
Correct Answer : C

Exp: Neutrophits containing damaged cell

LE cells are counterpart of hematoxylin bodies seen in renal biopsy specimen. T


a monocyte which has engulfed antigen antibody complex.

(Q.113) False positive VDRL test in SLE is due to

(a) ANA

(b) Anticardiolipin antibody

(c) Anti Sm

(d) Anti – histone

Your Response :
Correct Answer : B
Exp: Anticardiolipin antibody

Many types of antibodies are elaborated in patients suffering from systemic lup
erythematosus. Anticardiolipin abs are responsible for false positive VDRL.

(Q.114) Increases incidence of B cell lymphoma is observed in which of the following diseases?

(a) Rheumatoid arthritis

(b) SLE

(c) Sjogren syndrome

(d) Polymositis / dermatomyositis

Your Response :
Correct Answer : C

Exp: Sjogren syndrome

Sjogren syndrome causes predisposition for development of marginal zone lymp


Initially proliferatoion is polyclonal but later on it becomes monoclonal.

(Q.115) Which of the following neoplasms has been associated with the use of oral contraceptives?

(a) Breast cancer

(b) Ovarian cancer

(c) Endometrial cancer

(d) Hepatic adenoma


Your Response :
Correct Answer : D
Exp: Hepatic adenoma
Beginning with high- dose combination contraceptive pills used over 20 years ag
have been studied extensively for a possible association with neoplasia. There is
scant evidence from this experience that use of oral contraceptives increases th
any type of cancer. Actually, the progestational component of combination pills
progestin-only minipills) may confer a protective effect against carcinoma of the
and endometrium, and avoiding ovulation may decrease the risk of developing
carcinoma.

(Q.116) Which is not associated with jaundice in infancy?


(a) Hereditary fructose intolerance

(b) Maple syrup urine disease

(c) Galactosemia

(d) Crigler- Najjar syndrome

Your Response :
Correct Answer : B

Exp: Maple syrup urine disease

Jaundice is not seen in maple syrup urine disease. It is seen in rest of the three
conditions.

Jaundice during the first 24 hour of life should always be considered to be due t
erythroblastosis fetalis until proved otherwise. Septicemia and intrauterine infe
such as syphilis, cytomegalovirus and toxoplasmosisshould also be kept in mind
especially if there is an increase in plasma direct-reacting bilirubin.

Jaundice after the first 24 hour may be "physiologic. or may be due to septicem
hemolytic anemia. galactosemia hepatitis, congenital atresia of the bile ducts,
inspissated bile syndrome following erythroblastosis fetalis, syphilis herpes simp
congenital infections.

(Q.117) Jackson’s Sign is seen in?

(a) Paraplegia

(b) Cerebellar diseases

(c) Parkinsonism

(d) Hemiplegia

Your Response :
Correct Answer : D

Exp: (In hemiparesis) an observation that during quiet respiration the movement of t
paralyzed side of the chest may be greater than that of the opposite side. Howe
paralyzed side moves less under forced respiration.

(Q.118) Which is a conjugated vaccine?


(a) Hepatitis B

(b) Rubella

(c) Hemophilus influenzae b

(d) Pertussis

Your Response :
Correct Answer : C

Exp: Hemophilus influenzae b

Hib capsulpr polysaccharide induces production of antibodies in older children a


adults. However this is inadequate in children less than 18 months of age who f
achieve protective level of antibody or a long lasting immunological memory.

Hence Hib polysaccharide is conjugated to a T cell dependent protein antigen to


a new generation of highly effective vaccine.

Hib conjugate vaccine not only induces protective antibody and immunological
in infants but also results in decreased nasopharyngeal colonization of Hib. Thus
effect may be observed.

(Q.119) True about CADASIL

(a) Monogenic stroke syndrome

(b) White matter changes

(c) Onset is usually in the fourth or fifth decade of life

(d) All of the above

Your Response :
Correct Answer : D

Exp: All of the above

i. Complete basilar syndrome results in B/L long tract signs with variable cerebe
cranial nerve abnormal. Pt. will be comatose → High mid brain reticular activati
system.
ii. Locked – in syndrome → It is due to bilateral medial medullary syndrome, The
lesion is at medulla of oblongata. Preserved consciousness with quadriplegia &
12th cranial nerve palsy causing no tongue movement leading to patient in abilit
iii. CADASIL (cerebral Autosomal dominant arteriopathy with subcortical infarcts
leukoencephalopathy)

(Ref, H 18th Pg-3279)

It is a monogenic stroke syndrome.

It is an inherited disorder that presents as small-vessel strokes, progressive dem


and extensive symmetric white matter changes visualized by MRI.

Approximately 40% of patients have migraine with aura, often manifest as trans
motor or sensory deficits.

Onset is usually in the fourth or fifth decade of life.

This autosomal dominant condition is caused by one of several mutations in No


member of a highly conserved gene family characterized by epidermal growth f
repeats in its extracellular domain.

Other monogenic ischemic stroke syndromes are

i. Cerebral autosomal recessive arteriopathy with subcortical infarcts and


leukoencephalopathy (CARASIL)

ii. Hereditary endotheliopathy, retinopathy, nephropathy, and stroke (HERNS).

iii. Fabry's disease also produces both large-vessel arteriopathy and small-vesse
by an unknown mechanism.

(Q.120) Triple H therapy is done in?

(a) SAH

(b) SDH

(c) EDH

(d) Intraparenchymal blee(D.)

Your Response :
Correct Answer : A

Exp: (Ref. Harrison-18th/pg. 2262)

Vasospasm remains the leading cause of morbidity and mortality following


aneurysmal SAH.

Treatment with the calcium channel antagonist nimodipine improves outcome,


by preventing ischemic injury rather than reducing the risk of vasospasm.

Symptomatic cerebral vasospasm can also be treated by increasing the cerebral


perfusion pressure by raising mean arterial pressure through plasma volume ex
and the judicious use of IV vasopressor agents, usually phenylephrine or
norepinephrine.

Raised perfusion pressure has been associated with clinical improvement in ma


patients, but high arterial pressure may promote rebleeding in unprotected ane

Treatment with induced hypertension and hypervolemia generally requires mon


of arterial and central venous pressures; it is best to infuse pressors through a c
venous line as well.

Volume expansion helps prevent hypotension, augments cardiac output, and re


blood viscosity by reducing the hematocrit.

This method is called "triple-H" (hypertension, hemodilution, and hypervolemic


therapy.

(Q.121) Consider the following factors:

1.Head control

2.Social smile

3.Crawls

4.Sits up

The correct sequence in the developmental milestones during the first year of life is:

(a) 1,2,3,4

(b) 2,1,4,3

(c) 1,4,3,2

(d) 2,1,3,4

Your Response :
Correct Answer : B

Exp: Social smile is developed at 2 months of age. The infant intently regards the fac
mother by the age of 1 month.

The child recognizes the mother by the age of 3 month.


From 4 weeks-12 weeks the infant learns to lift and control his head in horizont
and then above the horizontal plane. However, in supine position the child gain
control between 12-20 weeks.

From the age of 5 months onwards the child learns to control his body in sitting
By the age of 8 months he can maintain a steady sitting position.

Pediatric milestones is a very important factual scoring opportunity in the entra


examination.however it is complicated by its varied timecourse described in diff
standard texts.

Here I have made an attempt of drawing a consensus from all standard texts an
providing a comprehensive list all together.you will have to mug it up.

GROSS MOTOR MILESTONES

3m Neck holding

d 4m Bidextrous reach

5m Sits with support

6m Sits in tripod position

8m Sits without support

9m Stands with support

10m Walks with support

11m Crawling and Pivoting

12m Stands without support

15m Walks without support

18m Explores drawers, running, walks backwards

24m Jumps, walking upstairs 2 feet/step

36m Rides tricycle, walks upstairs 1 feet/step and downstairs 2 feet/step

4 yrs Walks downstairs 1 feet/step

6 yrs Hops on both legs

FINE MOTOR MILESTONES


3- 4 m Grasp reflex disappears

4m Mouthing starts, Hand regard (11-20 weeks), bidextrous reach

5m Bidextrous grasp,

6m Transfers object from one hand to another, Unidextrous reach, palma

7m Unidextrous/Ulnar grasp

8m Radial grasp

9m immature pincer grasp

10m Feeds with much spilling

11m Mouthing stops

12m Mature pincer grasp

15m Feeds without much spilling, imitates, scribbles

18m Turns 2-3 pages at a time

2 yrs Turns 1 page at a time, puts on socks

2.5 yrs Threads a bead

3 yrs Dresses/undresses, Handedness develops, uses knife

4-5 yrs Ties shoelaces

LANGUAGE MILESTONES

1m Turns head to sound

2m Vocalizes

3m Cooing and babbling

4m Laughs aloud

6m Monosyllables

9m Bisyllables

12m Speaks 1st true word, repetition, knows 2 words with meaning
15-18m Jargon speech

18m Knows 10 words with meaning

2yrs Uses pronouns, points 1 part of body, speaks simple sentence with 3 w

3yrs Knows name and sex, shares toys,asks questions

4yrs Tells a story, plays co-operatively in a group,goes to toilet alone

5yrs Names 5 colors, helps in hosehold tasks

SOCIAL MILESTONES

1m Face regard-looks intently at mother

2m Social smile

3m Recognizes mother, anticipates feeds, shows interest in surroundings

6m Smiles at mirror image, likes and dislikes develop

7m Stranger anxiety, resists when toy is pulled

9m Wavws bye-bye, pulls mother to attract her

11m Holds arm out for shirt and feet for sleeves

12m Comes when name is called, understands phrases, plays simple ball ga

2yrs Asks for food, pulls people to show toys.

BOWEL AND BLADDER MELISTONES

1.5-2yrs Start toilet training

1.5 yrs Bowel control is achieved

1.5-2 yrs Dry by day

2 yrs 50 % dry by night

3 yrs 75 % dry by night

5 yrs 90 % dry by night

PAPER-WORK MILESTONES
18m Spontaneous scribbling

2yrs Copies horizontal line

3 yrs Draws a circle

4yrs Draws a rectangle or a plus sign

5yrs Draws a triangle or a tilted cross

6 yrs Draws a hexagon

7 yrs Draws a kite

8yrs Draws a double cross

9yrs Draws a cylinder

11 yrs Draws a cube.

PLAY MILESTONES

10m Plays peek-a-boo

1yr Egocentric pretend play

2yr Parallel play

4yr Plays with others

5yr Domestic play.

MISCELLANEOUS MILESTONES

3m Ocular fixation occurs

4m Binocular vision is established

10m Uncovers hidden toys

18m Rapproachment develops.

(Q.122) Which is true regarding a 15 Month old child?

(a) Crawl upstairs

(b) Make a tower of 3 cubes


(c) Name a familiar object

(d) All of the above

Your Response :
Correct Answer : D

Exp: A 15 month old child will be able to perform all the tasks as mentioned in the qu

A 15-month-old child will also be able to take several steps side ways. He will als
able to feed himself with a spoon without spilling its contents. He will also be ab
walk backwards.

(Q.123) You are attending a delivery. A baby weighing 3 kg is born and has not cried immediately.
What sequence of resuscitation steps will you carry out.

.Dry the baby; position the baby; tactile stimulation; suction of mouth and
(a) nose; free flow oxygen if required

.Dry the baby; free flow oxygen; position the baby; suction of mouth and
(b) nose

Position the baby; suction of mouth and nose; free flow oxygen; tactile
(c) stimulation; dry the baby

.Dry the baby; position the baby; suction of mouth and nose; tactile
(d) stimulation; free flow oxygen if required

Your Response :
Correct Answer : D
Exp: Steps of resuscitation (TABC)

T: Maintenance of temperature:

1. Radiant heat source.

2. Drying the baby.

3. Remove wet linen.

A: Establish an open airway:

1. Position the infant.

2. Suction the nose and mouth (trachea in some cases).


3. If required insert ET tube.

B: Start breathing:

1. Give tactile stimulation to start respiration.

2. Positive pressure ventilation if required by bag and mask or by bag and ET


tube.

C: Circulation:

1. Stimulate and maintain blood circulation with chest


compression &/or medication.

Drying the baby often provides tactical stimulation for crying.

The neonate should be placed on his back with the neck slightly extended.

After this first suctioning of mouth is done followed by suction of nose.

If the baby does not cry then tactile stimulation and oxygenation is required.

(Q.124) Daily requirement of calories and protein for a 10 kg weighing child are,
respectively:

(a) 800 kcal and 20 gm

(b) 1000 kcal and 30 gm

(c) 1000 kcal and25 gm

(d) 1200 kcal and 25 gm

Your Response :
Correct Answer B
:
Exp: 1000 kcal and 30 gm

Weight 10 kg means the child is about 1 year of age.

The energy requirement is roughly 112 kcal/kg and hence it is 10 )(


112 = 1120 kcal.

The protein requirement is 1.2 g/kg/day. Hence protein


requirement is 10 )( 1.2 = 12 g.

Hence the best answer is choice B.


Important points

1. The additional need for energy is 150 kcal/day during 1st


trimester and 350 kcal/day during 2nd and 3rd trimester.

2. During lactation additional energy requirement is 500 kcal/day.

3. Average adult requires 0.75 g/kg of protein/day.

4. During pregnancy additional protein intake of 1.3, 6.1 and 10.7


g/day is required in 1st, 2nd and 3rd trimester respectively.

5. During lactation additional protein requirement is 14.7 g/day


during first 6 months.

(Q.125) The following condition is not associated with an Anti-


phospholipid syndrome:

(a) Venous thrombosis

(b) Recurrent foetal loss

(c) Thrombocytosis

(d) Neurological manifestations

Your
Response :
Correct C
Answer :
Exp: Thrombocytosis:

Complications of antiphospholipid
syndrome:

a. Recurrent fetal loss b.


IUGR c. Pregnancy associated
hypertension

d. Placental abruption e. Recurrent


thrombotic events f. Thrombocytopenia.

(Q.126) Which of the following strategy can best prevent contrast-induced nephrotoxicity in a patient with a
renal function?
(a) N-acetyl cysteine

(b) Mannitol

(c) Fenoldopam

(d) Use of Low osmolar contrast agents

Your Response :
Correct Answer : D

Exp: Use of Low osmolar contrast agents.

Preventive measures have been proposed for contrast nephropathy:

Low osmolar contrast agents must be preferred.Q


It is clear that hydration is an effective preventive measure.Q
Other measures that have been proposed include:
N-acetylcysteine:Q
In contrast, N-acetyl- cysteine appears to be safe, and its use in patients at high
radiocontrast nephropathy is reasonable, based on its low side effect profile.
Theophylline and aminophylline (adenosine antagonists):
offer the potential advantage of use immediately preceding radiocontrast
administration, although the benefit, if present, appears marginal in most studie
Sodium bicarbonate.
Volume expansion with bicarbonate-containing IV fluids has been suggested to
superior to sodium chloride (saline) administration and showed a significant ben
Unlike N-acetylcysteine, the use of sodium bicarbonate does not obligate a dela
imaging. Whether a combination of strategies (e.g., N-acetylcysteine + sodium
bicarbonate) offers additive benefit and that patients require treatment remain
and warrant further study.
Loop diuretics and mannitol:Q
insufficient evidence to support the use of loop diuretics or mannitol to prevent
radiocontrast nephropathy or any other cause of ARF.
Dopamine:
Likewise, despite its widespread use, dopamine has proved ineffective as a prop
agent.
Fenoldopam [dopamine alpha-1 specific agonist]:Q
approved for use as a parenteral antihypertensive agent, has been tested in sev
clinical trials and does not appear to reduce the incidence of contrast nephropa
Moreover, fenoldopam is associated with significant side effects, including sys
hypotension, and its use as an agent to prevent radiocontrast nephropathy sh
discouraged.
(Q.127) Which of the following is the most likely adverse response to occur as a result of vincristin?

(a) Nephrotoxicity renal dysfunction or failure

(b) Neutropenia

(c) Peripheral sensory and motor neuropathy

(d) Pulmonary damage

Your Response :
Correct Answer : C

Exp: Peripheral sensory and motor neuropathy

Vincristine is one of relatively few cytotoxic anticancer drugs that does not caus
marrow suppression (and all the potential consequences of that) as its main tox
Rather, it causes neuropathies involving both sensory and motor nerves. Parest
are a common example of the former (hearing loss can also occur); muscle wea
and obtunded reflexes are examples of the latter. Important note: Vincristine d
from theother two vinca alkaloids, vinblastine and vinorelbine, which do cause b
marrow suppression (and not neuropathies) as their main doselimiting toxicity

(Q.128) Antitubercular drug which does not cross blood brain barrier is:

(a) Streptomycin

(b) INH

(c) Rifampicin

(d) Pyrazinamide

Your Response :
Correct Answer : A
Exp: Streptomycin

Streptomycin penetrates tubercular cavities, but does not cross the


blood brain barrier.

(Q.129) All of the following drugs reduce afterload, EXCEPT:

(a) Nitroglycerine
(b) Dopamine

(c) Hydrallazine

(d) Sodium nitroprusside

Your Response :
Correct Answer : B

Exp: Dopamine

Dopamine is a vasopressor agent and thus increases the afterload esp at


higher rates of infusion .

(Q.130) Most serious side effect with the use of aminoglycosides is:

(a) Ototoxicity

(b) Nephrotoxicity

(c) Bone marrow suppression

(d) Anaphylaxis

Your Response :
Correct Answer : A

Exp: Ototoxicity

The most serious dose and duration related adverse effect of aminoglycosides is
ototoxicity. Other important side effects of aminoglycosides are nephrotoxicity
neuromuscular blockade. Nephrotoxicity is totally reversible , if drug is immedia
discontinued .

(Q.131) Which of the following drug should not be stopped abruptly:

(a) Theophylline

(b) Chlorpheniramine

(c) Corticosteroids

(d) Hydralazine

Your Response :
Correct Answer : C

Exp: Corticosteroids

Corticosteroids should not be stopped abruptly because with prolonged steroid


hypothalamo-pituitary-adrenal axis suppression occurs and the patient may dev
acute adrenal insufficiency on sudden withdrawal of the drug.

• During infection in a patient on steroid therapy, steroids should not be withdr


despite the immune suppressant effect of these drugs because infection is a sta
stress and abrupt withdrawal of steroid therapy will precipitate acute adrenal
insufficiency.

(Q.132) Epinephrine is most useful in:

(a) Bronchial asthma

(b) Anaphylactic shock

(c) Peripheral vascular disease

(d) Wide angle glaucoma

Your Response :
Correct Answer : B

Exp: Anaphylactic shock

• Epinephrine, being the physiological antagonist of histamine, affords quick rel


anaphylaxis and angioedema.

(Q.133) Which of the following drugs is useful in acute attack of gout:

(a) Furosemide

(b) Sulfinpyrazone

(c) Allopurinol

(d) Piroxicam

Your Response :
Correct Answer : D

Exp: Piroxicam
• Drugs used for treatment of acute gout

1. NSAIDs - response is slower than with colchicine but are better tolerated. Piro
naproxen, indomethacin and phenylbutazone have been found useful.

2. Colchicine - specifically suppresses gouty inflammation.

3. Corticosteroids - can be given as intra articular injection.

Drugs used for treatment of chronic gout:

1. Probenecid - uricosuric agent

2. Sulfinpyrazone - uricosuric agent

3 Allopurinol - xanthine oxidase inhibitor (inhibits synthesis of uric acid)

(Q.134) Locally acting aminoglycoside is:

(a) Streptomycin

(b) Neomycin

(c) Netilimycin

(d) Sisomycin

Your Response :
Correct Answer : B

Exp: Neomycin

• Neomycin is a topically and orally effective aminoglycoside, orally also it acts


through local action

• Oral neomycin and lactulose are used in hepatic coma.

• Neomycin is used orally for preparation of bowel before surgery.

(Q.135) False about celiac sprue is:

(a) Brush border enzymes are increased


(b) Intolerance is found to Barley, Rye, etc

(c) Antigliadin, anti-reticulin antibodies positive


(d) Increased risk of small intestinal malignancy
Your Response :
Correct Answer : A
Exp: (Brush border enzymes are increased) (Ref: Harrison-18th Pg- 2469)
Coeliac disease

It results from dietary gluten intolerance.

Intolerance is found to gluten containing food materials


like Barley, Rye, Oats, Wheat, etc

Age of onset 20-40 years

Strong association with HLA-DR3, DQw2, dermatitis herpetiformis.

IgA entiendomysial antibodies and antibodies against tissue glutaminase provide


supporting evidence.

Histologically it is characterized by small intestinal villous atrophy.

Increased risk of malignancy: T-cell lymphoma, small bowel carcinoma and


squamous cell carcinoma of esophagus.

Celiac sprue IgA anti-gliadin

IgA anti endomysial

IgA anti tissue transglutaminase

IgA anti reticulin antibody

(Q.136) Loading dose of a drug is given:

(a) When half-life of a drug is long

(b) When therapeutic index is low

(c) When drug follows first order kinetics

(d) When serum concentration is to be achieved rapidly

Your Response :
Correct Answer : D
Exp: When serum concentration is to be achieved rapidly

• Loading dose of a drug is given when target plasma concentration


is to be achieved rapidly.
• Loading dose is calculated as:

Loading dose =

Target plasma concentration X Volume of distribution


Fraction of the dose reaching systemic circulation in active form

(Q.137) Broca’s area of speech is located in:


(a) Superior temporal gyrus

(b) Parietal lobe

(c) Inferior gyrus of frontal lobe

(d) Superior gyrus of frontal lobe

Your Response :
Correct Answer : C

Exp: Inferior gyrus of frontal lobe

BROCA’S AREA IS the area of the left cerebral hemisphere at the POSTERIOR EN
INFERIOR FRONTAL GYRUS. It contains the motor speech area and controls mov
of tongue, lips, and vocal cords.

(Q.138) Taste buds responsible for carrying bitter taste sensation are located at:
(a) Posterior aspect of the tongue

(b) At the tip of the tongue

(c) Just behind the tip of the tongue

(d) At the sides of the tongue

Your Response :
Correct Answer : A

Exp: Posterior aspect of the tongue

BASIC TASTES AREAS OF TONGUE

• Sweet At the tip of the tongue

• Sour Along the edges of the tongue


• Bitter On the back of the tongue

• Salt On the anterior dorsum of the


tongua

(Q.139) Which of the following is true regarding Bainbridge reflex:


(a) Is not a true reflex

(b) Rapid infusion of fluids leads to increase in heart rate


(c) Abolished by bilateral vagotomy

(d) Due to response to local stretch

Your Response :
Correct Answer : C

Exp: Abolished by bilateral vagotomy

BAINBRIDGE REFLEX: Rapid infusion of blood or saline produces a rise in heart ra


initial heart rate is low. It is a true reflex rather than a local response.

(Q.140) A 30-year-old male has diarrhea, anemia, and raised liver enzymes. Which of the following will you d
further evaluation?

(a) Anti-thyroid antibodies

(b) Anti-smooth muscle antibodies

(c) Anti-mitochondrial antibodies

(d) Anti-endomysial antibodies

Your Response :
Correct Answer : D

Exp: Anti-endomysial antibodies (Ref: Harrison-18th Pg- 2469)

The symptoms of celiac sprue may appear with the introduction of cereals in an
diet, although there is frequently a spontaneous remission during the second de
life that may be either permanent or followed by the reappearance of symptom
several years.

Alternatively, the symptoms of celiac sprue may first become evident at almost
throughout adulthood. In many patients, frequent spontaneous remissions and
exacerbations occur. The symptoms range from significant malabsorption of mu
nutrients, with diarrhea, steatorrhea, weight loss, and the consequences of nutr
depletion (i.e., anemia and metabolic bone disease), to the absence of any
gastrointestinal symptoms but with evidence of the depletion of a single nutrien
iron or folate deficiency, osteomalacia, edema from protein loss). Asymptomati
relatives of patients with celiac sprue have been identified as having this disease
by small-intestinal biopsy or by serologic studies [e.g., antiendomysial antibodie
transglutaminase (tTG)]. The availability of these "celiac serologies" has led to a
substantial increase in the diagnosis of celiac sprue and the diagnosis is now bei
primarily in patients without "classic" symptoms but with atypical and subclinica
presentations.

Asymptomatic relatives of patients with celiac sprue have been identified as hav
disease either by small-intestinal biopsy or by serologic studies (e.g., antiendom
antibodies).

As in patients with celiac disease, dietary gluten sensitivity in patients with Derm
Herpetiformis is associated with IgA anti-endomysial autoantibodies that target
transglutaminase.

(Q.141) Streptokinase produce thrombolysis after binding to which of the following proteins?
(a) Antithrombin III
(b) Fibrin
(c) Plasminoge
(d) Protein C
Your Response :
Correct Answer : C
Exp: Plasminogen
The fibrinolytic activity of streptokinase is due to its ability to bind and cleave
plasminogen, producing plasmin. Plasmin directly cleaves fibrin, both between a
within the fibrin polymers, thus breaking up thrombi and potentially restoring b
flow to ischemic cardiac muscle. This same mechanism of fibrinolysis is shared b
urokinase and tissue-plasminogen activator (tPA). Antithrombin III is a coagulati
inhibitor that binds to and inactivates thrombin. Antithrombin III is anticoagulan
fibrinolytic. Fibrin is not directly acted upon by streptokinase. It is indirectly clea
through the action of plasmin. Protein C is a glycoprotein that modulates coagu
inhibiting the procoagulant activities of factors V/Va and VIII/VIIIa. Protein C has
inherent fibrinolytic activity. Thrombomodulin is an anticoagulant protein that b
thrombin and diminishes its capacity to activate fibrinogen, Factor V, and platel
Thrombomodulin has no fibrinolytic activity.

(Q.142) Primary motor area for shivering is:


(a) Dorsomedial posterior hypothalamus
(b) Ventromedial anterior hypothalamus

(c) Red nucleus

(d) Motor cortex

Your Response :
Correct Answer : A

Exp: Dorsomedial posterior hypothalamus

Primary motor center for shivering is located in the DORSOMEDIAL PORTION OF


POSTERIOR HYPOTHALAMUS near the wall of third ventricle. This area is excited
signals from skin and spinal cord and inhibited by signals by heat centers from
hypothalamus. During maximum shivering body heat production raises about fiv
the normal.

(Q.143) Which of the following is a pain sensitive intracranial structure:


(a) Pia mater

(b) Dura mater

(c) Pial veins

(d) Brain

Your Response :
Correct Answer : B

Exp: Dura mater

STRUCTURES SENSITIVE TO PAIN STRUCTURES INSENSITIVE TO PAIN

• SCALP AND APONEURO11CA • VENTRICULAR EPENDYMA

• MIDDLE MENINGEAL ARTERY • CHOROID PLEXUS

• DURAL SINUSES • PIAL VEINS

• FALX CEREBRI • MUCH OF THE BRAIN PARENCHYMA

• PROXIMAL SEGMENTS OF THE LARGE PIAL ARTERIES

(Q.144) A patient has a painful ulcer on the tip of his tongue. Which of the following cranial nerves carries th
sensation he experiences?
(a) V2
(b) V3
(c) VII
(d) IX
Your Response :
Correct Answer : B
Exp: V3
The innervation of the tongue is complex. The mandibular division of the trigem
nerve (V3) carries general somatic sensation from the anterior two-thirds of the
The maxillary division (V2) carries somatic sensation from the palate, upper gum
upper lip. The facial nerve (VII) carries taste from the anterior two-thirds of the
The glossopharyngeal nerve (IX) carries sensation and taste from the posterior o
of the tongue. The vagus nerve (X,) carries sensation from the lower pharynx.
(Q.145) In CNS the phagocytosis is done by:
(a) Oligodendroglia

(b) Microglia

(c) Astrocytes

(d) All of the above

Your Response :
Correct Answer : B

Exp: Microglia

The reticuloendothelial system comprises monocyte-derived phagocytic cells tha


lymph nodes and several other

organs. The reticuloendothelial system comprises monocyte-derived phagocytic


that are lymph nodes several other organs.

• LIVER KUPFFER CELLS

• LUNG ALVEOLAR MACROPHAGES

• SPLEEN, KIDNEY MESANGIAL CELLS

• BRAIN MICROGLIA

(Q.146) Which of the following carries conscious proprioception:


(a) Spinocerebellar tract

(b) Ponto cerebral tract


(c) Anterior spinothalamic tract

(d) Dorsal column

Your Response :
Correct Answer : D

Exp: Dorsal column

The proprioceptive impulses are transmitted up the spinal cord in the dorsal col
Much of these inputs go to the cerebellum but some pass to the cerebral cortex
medial lemnisci and thalamic radiations.

(Q.147) Infectious diseases in which ISOLATION is important part of


disease management:

(a) Cholera

(b) Typhoid

(c) Polio

(d) Hepatitis

Your
Response :
Correct A
Answer :
Exp: (Cholera)

Infectious diseases in which ISOLATION is vital:

· Diphtheria, · Plague

· Cholera · Some Streptococcal infections

(Q.148) Both active and passive vaccination can be given


together in all EXCEPT:

(a) Diphtheria

(b) Tetanus

(c) Rabies
(d) Hepatitis A

Your
Response :
Correct D
Answer :
Exp: (Hepatitis A)

Both active and passive vaccination can be


given together in

Diphtheria, Tetanus, Rabies and Hepatitis B.

(Q.149) Iceberg phenomenon is not seen in:

(a) Measles

(b) AIDS

(c) Polio

(d) Rubella

Your Response :
Correct Answer : A

Exp: (Measles)

Iceberg of disease:

Disease in a community can be compared with an iceberg.

The “floating tip” of iceberg _ represents what the physician sees in the commu
(Clinical cases).

The vast “submerged” portion _ represents the hidden mass of disease (latent
inapparent, pre-symptomatic, undiagnosed cases and carriers).

The “water-line” _ represents the demarcation between apparent and inappare


diseases.

The “hidden” part _ represents an important undiagnosed reservoir of infection


disease in the community, and its detection and control is challenge to modern
techniques in PSM. In some diseases

(e.g. hypertension, DM, anaemia, malnutrition, mental illness) the unknown mo


(submerged part of iceberg) far exceeds the known morbidity.

The “bottom” of iceberg _ one of the major deterrents in study of chronic disea
unknown etiology in absence of methods to detect the subclinical state.

(Q.150) In an area not covered by measles immunization, the attack rate of measles is:

(a) 70%

(b) 80%

(c) 90%

(d) 100%

Your Response :
Correct Answer : B

Exp: (80%)

Eradication of measles:

It is believed that measles, like smallpox, is amenable to eradication, especially


only one dose of measles vaccine is needed and now more heat stable measles
has developed. But in area not covered by measles immunization, the attack rat
One attack of measles generally confers lifelong immunity (second attacks are r

WHO’s measles elimination strategy comprises 3 part vaccination strategy:

1. Catch-up (targeting all children between 9 months to 14 years irrespective of


of measles or vaccination status)

2. Clean-up (routine services aimed at vaccinating more than 95% of each succe
birth cohort)

3. Follow-up (subsequent nation wide vaccination campaign conducted every 2


targeting all children born after the catch-up campaign)

(Q.151) Which disease does not occur as seasonal variation?

(a) Measles

(b) Rubella

(c) Gastroenteritis
(d) None

Your Response :
Correct Answer : D

Exp: None of the above

Measles is a winter disease because people crowd together indoors.

Rubella usually occurs in a seasonal pattern i.e. in temperate zones during the la
winter and springs.

The seasonal variation of the meningococcal meningitis is well established, out-


occur more frequently in dry and cold months.

Distinct seasonal pattern of diarrhea occur in many geographical areas. In temp


climates, bacterial diarrhea occurs more frequently during warm season, while
diarrhea especially by Rotavirus peak during winter.

(Q.152) The interstitial cells of Cajal:

fire action potentials to initiate the electrical slow waves propagating along th
(a) tract.

(b) Are found between the longitudinal and circular smooth muscle layers.

(c) Hormonally regulate peristalsis.

(d) These are cell of origin for GI lymphomas.

Your Response :
Correct Answer : B

Exp: Are found between the longitudinal and circular smooth muscle layers.

The interstitial cells of Cajal, which are found between the longitudinal and circu
smooth muscles, are thought to be involved in the generation of slow waves. Th
are involved in the generation of slow waves, which trigger action potentials.

These are cell of origin for GI leomyomas (GIST).

(Q.153) Which of the following is true about Meningococcal meningitis?

(a) Case fatality less than 10% in untreated cases

(b) Cases are the main source of infection


(c) 0.4% children are of 10 years age

The meningococcal vaccine is not recommended for use in infants and children
(d) years of age

Your Response :
Correct Answer : D

Exp: (The meningococcal vaccine is not recommended for use in infants and childre
2 years of age)

Meningococcal meningitis (cerebrospinal fever)

It is an acute communicable disease caused by gram-negative diplococci, N.


meningitidis.

The fatality of typically untreated case is about 80%.

With early diagnosis and treatment, case fatality rates have been declined to les
10%.

The zone lying between 5 and 15 degree N of the equator in tropical

Africa is called the “meningitic belt”.

Groups A and C and to some extent Group B meningococci are capable of causin
epidemic.

Carriers are the most vital source of infection.

Clinical cases present only a negligible source of infection.

It is predominantly a disease of children and young adults of both sexes.

Seasonal variation is well-established dry and clod months.

The disease spreads mainly by droplet infection.

The portal of entry is nasopharynx.

A powerful antibiotic like Rifampicin is needed to eradicate the carrier state


(chemoprophylaxis).

Penicillin is drug of choice for treatment of disease.

Effective vaccines prepared from purified Group A, C, Y, and/or W135 meningoc


polysaccharides are now available.

The meningococcal vaccine is not recommended for use in infants and children
years of age.

The vaccine is contraindicated in pregnant women.

(Q.154) Murine typhus is transmitted through which of the following?

(a) Mite

(b) Tick

(c) Rat

(d) Flea

Your Response
:
Correct D
Answer :
Exp: (Flea)

Arthropod-borne diseases:

——————————————————————————————

Arthropod Diseases transmitted

——————————————————————————————

1.Housefly: Typhoid and paratyphoid fever

Diarrhea, dysentery

Cholera

Gastroenteritis

Amoebiasis

Helminthic infestations

Poliomyelitis

Conjunctivitis

Trachoma

Anthrax,
Yaws, etc.

2. Sandfly: Kala-azar

Oriental sore

Sand-fly fever

Oraya fever.

3. Tsetse fly: Sleeping sickness

4. Louse: Epidemic typhus

Relapsing fever

Trench fever, and

Pediculosis

5. Rat flea: Bubonic plague

Endemic typhus

6. Black fly: Onchocerciasis

7. Reduviid bug: Chagas disease

9. Hard tick: Tick typhus

Tick paralysis

Viral encephalitis

Viral hemorrhagic fevers (e. g. KFD)

Tularemia

Babesiosis

10. Soft tick: Q fever

Relapsing fever

11. Trombiculid Scrub typhus mite: Rickettsial pox

12.Itch mite: Scabies (sarcopti scabi)

13.Cyclopes: Guinea worm disease


Fish tapeworm (D. latus)

14.Cockroaches: Enteric pathogens

——————————————————————————————

(Q.155) It is true regarding Endemic typhus that:

(a) Man is the only reservoir of infection

(b) Flea is a vector of the disease

(c) The rash developing into eschar is a characteristic presentation

(d) Culture of the etiological agent in tissue culture is diagnostic modality

Your Response :
Correct Answer : B

Exp: (Flea is a vector of the disease)

Rickettsial diseases:

—————————————————————————————

Disease Caused by Vector of transmission

——————————————————————————————

Epidemic typhus R. prowazekii Louse

Endemic typhus R. typhi Rat flea

RMSF R. rickettsii Tick

Rickettsial pox R. akari Mite

Scrub typhus R. tsutsugamushi Mite

Trench fever Roxiella Quintana Louse

Q fever Coxiella burnetti Tick

—————————————————————————————

Rickettsial diseases is a very frequently asked topic in exams.Here is a tell-all t

DISEASE CAUSATIVE INSECT VECTOR RESERVIOR WEIL- NEI


FELIX REA
ORGANISM TEST REA

Epidemic R.prowazeki Human louse Humans ++ --


typhus/Red OX
louse Pediculus humanis 19
disease/Gaol corporis/capitis.
fever/Brill
zinsers
disease

Endemic R.typhi Rat flea – Xenophelus Rodent ++ ++


typhus / (mooseri) cheopsis OX
Murine 19
typhus Cat flea – Ctenocephalus
felis

Scrub typhus R. Trombiculid mite Rodent ++ --


/ Chigger OX K
borne tsutsugamushi Leptotrombidium
typhus deliensis – India.

L.akamushi – Japan

Indian tick R.conorii Tick – Rhipicephalus Rodents,Dogs -- ++


typhus/ sanguinis,Haemophysalis
leachi,Amblyomma and
Fever Hyalloma ticks
boutonessi

RMSF R.rickettsiae Tick – Dermatocentor Rodents ++ --


andersoni OX
19
and
OX 2

Rickettsial R.akari Mite – Gamasid mite – Mice -- ++


pox Liponyssoides sanguines

Q fever Coxiella NIL (airborne cattle -- --


burnetti transmission)
Trench Rochiamlae Human louse Humans -- --
fever/ Five quintana
day fever Pediculus humanis
corporis/capitis

(Q.156) Beijing P3 strain is used for vaccination against ?

(a) Varicella

(b) Rubella

(c) Japanese encephalitis

(d) Cholera

Your Response :
Correct Answer : C
Exp: JE

Here is comprehensive list of all the specific strains used for vaccination :

VACCINE STRAIN USED

BCG Danish 1331

Measles Edmonston-Zagreb strain

Schwatz strain

Moraten strain

Mumps Jeryll Lynn strain

RIT 4385

Leningrad 3 strain

L-Zagren strain

Urabe strain

Rubini strain

Rubella RA 27/3 strain


HPV 77 strain

Cendehill strain

Yellow fever 17 D strain

Asibi strain

Dakar strain

Varicella OKA strain

Japanese encephalitis Nakayama strain – MC

Beijing P3 strain

SA 14-14-2 strain

Malaria SPf 66 strain {Lytic cocktail}

Pf 25 strain

Brucella Brucella abortus 19 BA strain

Typhoral TY21a

HIV mVA {modified vaccinai Ankara }


strain

rAAV {recombinant adeno associated


viral} strain

CTL {cytotoxic Tlymphocyte strain}

AIDSVAX strain

Subunit vaccine strain

Pnemococcal PPV 23{Unconjugated vaccine}

PCV 7{conjugated vaccine}

Plague Recombinant vaccine using F1 and V


antigens

Anthrax Mazzuchi/Sterne vaccine

Rickettsia Weigl’s vaccine{R.prowazeki}


Castaneda formalinized mouse lung
vaccine

Live attenuated strain E vaccine

Cox inactivated yolk sac vaccine

HPV HPV-VLT vaccines

Gardasil – quadrivalent {6,11,16,18}

Cervarix – bivalent {16,18}

Cholera Whole Cell killed with or without


nontoxic beta subunit

Live attenuated- Strain CVD 103 HgR.

(Q.157) Lice are not the vectors of:

(a) Relapsing fever

(b) Q fever

(c) Trench fever

(d) Epidemic typhus

Your Response :
Correct Answer : B
Exp: (Q fever)

Louse-borne infections include:

1. Pediculosis

2. Epidemic typhus

3. Relapsing fever

4. Trench fever

Q fever is caused by Coxiella burnetii and vector of transmission is soft


tick.

(Q.157) Lice are not the vectors of:


(a) Relapsing fever

(b) Q fever

(c) Trench fever

(d) Epidemic typhus

Your Response :
Correct Answer : B

Exp: i.e. supine posture is better than erect posture

Haemothorax

Blood accumulate in pleural cavity following injury to chest

Breath sound are decrease on the side of injury

Chest x-ray obtained in supine position may reveal accumulation of blood great
200ml. . Pleural space may accumulate upto 3L of blood

Treatment

Chest tube placement (36 French) (Bleeding stops as lung is re-expanded)

Thoractomy is needed in few patients only who continue to bleed.

(Q.159) Which of the following is most appropriately termed as penetrating neck injury?

(a) 2cm deep

(b) Injury of vital structure

(c) Breach of platysma

(d) Through and through wound

Your Response :
Correct Answer : C

Exp: i.e. Breach of Platysma (Ref Sabiston 17le 489; Schwartz 81e 139)

Injuries that do not penetrate the platysma can be considered superficial and no
investigation is needed.

Wounds that penetrate the platysma must be further evaluated.


Penetrating Neck injury
Neck contains vital structure with little bony protection. Most severe neck injur
caused by penetrating wound and may present an immediate threat to life.

Platysma and sternomastoid are useful anatomic boundaries in determining sev


injury as all the major vascular and aerodigestive tract are located deep to platy
anterior to sternomastoid. Thus injury that penetrate platysma and anterior to
sternomastoid possess high risk.

Injury posterior to sternomastoid has low risk of bleeding but they might be ass
with spinal injury.

In craniocaudal direction neck can be divided in to:

- Zone I: Extends from sternal notch to cricoid cartilage. Injury in this zone caries
mortality.

- Zone II : Midportion of neck extends from cricoid cartilage to angle of mandibl


in this zone are usually manageable.

- Zone III : Extends from angle of mandible to base of skull. Surgical exposure is
in this zone.

. Resuscitative priority:

Airway> Bleeding> Definitive treatment.

Management:

Definitive Therapy

Vascular Injury:

Arterial injury should be debrided and repaired primarily, if possible. If primary


not practical due to loss of length a short interpositoned graft (Vein or
polytetrafluroethylene) should be used.

Major injuries of ECA can be treated by ligation, however ligation of ICA and CCA
be done only for uncontrollable hemorrhage or if repair is technically impossible

Airway injury:

Tracheal injury should be debrided and closed primarily. If there is significant tis
trachea can be mobilized sufficiently to allow loss of two tracheal rings. Loss of
proportion require tracheostomy and complex reconstructive procedure.

Laryngeal injury: Laryngeal injuries are treated by closure of mucosal laceration


reduction of cartilaginous fracture.

Pharynx and oesophagus:

- Injury to oesophagus present a difficult problem, if the diagnosis is made early


surgical repair is generally possible. If the diagnosis is delayed for more than 24
diversion and drainage is preferred.

(Q.160) Complication of empyema are all except:

(a) Empyema necessitans

(b) Bronchopleural fistula

(c) Osteomyelitis

(d) Pneumonia

Your
Respons
e:
Correct D
Answer :
Exp: i.e. pneumonia

Complication of empyema are

1. Empyema necessitansl

2. Chronic empyerna

3. Osteomyelitis

4. Chondritis of ribs or vertebrae

5. Pericarditis

6. Mediastinitis

7. Bronchopleural fistula

8. Disseminated infection of CNS

(Q.161) Muscle not cut in posterolateral thoracotomy is

(a) Serratus anterior


(b) Lattisimus dorsi

(c) Rhomboid major

(d) Pectoralis major

Your Response :
Correct Answer : D

Exp: i.e. pectoralis major

Posterolateral thoracotomy is most frequent thoracic incision used.

Muscle encountered and cut in posterolateral thoracotomy form posteriorly to


anteriorly

Trapezius

Latissimus dorsi

Serratus anterior

Note- Latissimus dorsi is divided as low as possible to minimize denervation

Posteriorly the rhomboid which lies deep to trapezius are divided to expose the
subscapular fascia

Serratus anterior is only separated over the fifth and sixth rib.

(Q.162) Regarding pectus excavatum all are true except:

(a) Gross CVS dysfunction

(b) Decrease in lung reserves

(c) Cosmetic deformity

(d) Depression chest

Your Response :
Correct Answer : A

Exp: Pectus Excavatum

Also called funnel chest or sunken chest

Most common chest wall deformity


It occur in 1/400 children

More common in males than females

Familial in 30% cases

Sternum is depressed with dish shaped deformity of the anterior portion of ribs
or both sides

Depression is deeper on right side than the left causing rotation of sternum.

Associated with

Musculoskeletal abnormalities such as scoliosis

Marfan's syndrome

Congenital heart disease

Presentation

Mostlyasymptomatic

Decrease in respiratory reserve

Pain along the costal cartilages with exercise

Occasionally palpitation or murmur is noted

Decrease stroke volume and cardiac output

Decrease maximal breathing capacity

Investigation

X-ray chest

CT scan

Indication for operative intervention

1. Cosmesis

2. Psychosocial factors

3. Presence of respiratory or cardiovascular insufficiency

Time of surgery - best results reported between 2 and 5 year of age

Things to remember
Pectus carinatum (pigeon chest)

In this condition the sternum is elevated above the level of the ribs

Treatment- for cosmetic reasons.

(Q.163) Most useful intravenous fluids in burn patients is

(a) Ringer lactate

(b) Normal saline

(c) Plasma expander

(d) Blood

Your
Response :
Correct A
Answer :
Exp: i.e. ringer lactate

Fluids used during resuscitation of bum patient.

Ringer lactate solution without dextrose (fluid of choice in


adult)

5% dextrose ringer lactate (fluid of choice in children)

Other crystalloid can also used,

Note: It is still in debate whether to use colloids in first 24


hour or not.

Use of albumin during iv resuscitation is not


recommended.

(Q.164) Couinaud Segment IX of liver belongs to:

(a) Left anatomical lobe

(b) Left surgical lobe

(c) Right surgical lobe

(d) Right paracaval portion of posterior liver.


Your Response :
Correct Answer : D

Exp: REF : Hepatic surgical anatomy Surg Clin N Am 84 (2004) 413–435

Couinaud’s liver segmentation

The Couinard segmentation (Fig. 6C, D) system is based on the distribution in th


both the portal vein and the hepatic veins *26+ and shows a specific consideratio
the caudate lobe. Fissures of the three hepatic veins (portal scissurae) divide th
into four sectors (segments), lateral and paramedian, on the right and left sides
respectively. The planes containing portal pedicles are called hepatic scissurae.
segments are described, one for the caudate lobe (segment I), three on the righ
(segments II, III, and IV), and four on the left (segments V, VI, VII, and VIII). In ge
the segments of this classification correspond to subsegments of Healey and Sch
*19+. Couinaud’s system of liver segmentation differs from Healey and Schroy’s
system in several ways, however. According to Couinaud [22,26], a subdivision o
segment IV and the caudate lobe into two parts is not justified. Furthermore, Co
asserted that a study of organogenesis and comparative anatomy suggests that
umbilical fissure is the hepatic scissura between segments III and IV *22+. For He
Schroy *19+, however, the umbilical fissure is the plane of separation between t
of biliary (and consequently portal vein) branches between the medial and later
segment of the left lobe [26]. At the close of the last century, several investigato
including Couinaud and coworkers, used the term segment IX for an area of the
sector of the liver close to the IVC [29–32]. In 2002, however, Abdalla, Vauthey
Couinaud *33+ wrote, ‘‘Because no separate veins, arteries, or ducts can be defin
the right paracaval portion of the posterior liver and because pedicles cross the
proposed division between the right and left caudate, the concept of segment I
abandoned.’’ The genesis and death of segment IX is found in articles by Couina
other investigators

(Q.165) Commonest hypospadias is

(a) Penile

(b) Glandular

(c) Scrotal

(d) A or C

Your Response :
Correct Answer : B
Exp: i.e. glandular

Hypospadias results from incomplete fusion of the urethral plate during develop
the male penis. Hypospadias occurs in one in 300 males. The risk for hypospadia
increased by history of maternal estrogen or progestin use during pregnancy.
Hypospadias are classified by the location of the urethral opening. Approximate
hypospadias occur on the corona or distal shaft of the penis. Neonates with a
hypospadias are not at increased risk for having other congenital abnormalities
urinary tract. However, penoscrotal or perineal hypospadias may represent an i
disorder and evaluation should include a karyotype. An intersex work-up is also
indicated if a hypospadias and an undescended testicle are noted.

(Q.166) All are true about Peyronie's disease except

(a) Self limiting

(b) Medical treatment is effective

(c) Association with Dupuytren's contracture

(d) Calcified plaques

Your
Response :
Correct D
Answer :
Exp: i.e. calcified plaque

It is fibrous plaque present on the dorsal surface


of the penis.

(Q.167) A sebaceous gland, secreting smegma is known


as

(a) Cowper's gland

(b) Bartholins gland

(c) Tyson gland

(d) Brenner's gland

Your
Respons
e:
Correct C
Answer :
Exp: i.e. Tyson's gland

(Q.168) The treatment of cecal volvulus is:

(a) Observation alone

(b) Colonoscopic detorsion, Bowel prep. And elective resection

(c) Operative detorsion & cecopexy

(d) Operative detorsion, right hemicolectomy with ileocolostomy

Your Response :
Correct Answer : D

Exp: Operative detorsion, right hemicolectomy with ileocolostomy

Cecal volvulus results from non fixation of the right colon. Rotation occurs aroun
ileocolic blood vessels and vascular impairment occurs early. Plain x-ray of the a
shows characteristic kidney-shaped, air filled structure in the left upper quadran
(opposite the site of obstruction), and a Gastrografin enema confirms obstructio
level of the volvulus.
Unlike sigmoid volvulus, cecal volvulus can almost never be detorsed endoscopi
Moreover because vascular compromise occurs early in the course of cecal volv
surgical exploration is necessary when the diagnosis is made.
Right hemicolectomy with primary ilecolic anastomosis can be performed safely
prevents recurrence. Simple detorsion or detorsion and cecopexy are associated
high rate of recurrence.
(Q.169) Standard therapy for malignant pleural effusions is?

(a) Repeated thoracentesis with injection of a sclerosing agent

(b) Radiation therapy

(c) Thoracotomy with pleurectomy

(d) Tube thoracostomy with subsequent instillation of a sclerosing agent

Your Response :
Correct Answer : D

Exp: Tube thoracostomy with subsequent instillation of a sclerosing agent


Malignant pleural effusion is a very common presenting complaint for patients
variety of malignancies. Repeated thoracentesis is quite uncomfortable for the p
and very rarely effects pleurodesis. Thoracotomy with pleurectomy is a major o
and although it would likely effect pleurodesis, the risk is usually prohibitive. Ra
therapy is occasionally indicated in patients who have a chylothorax with very p
drainage of large amounts of fluid. Vigorous diuresis with infusion of albumin w
drain the cavity. The standard therapy, then, is tube thoracostomy with subsequ
instillation of sclerosing agents. There have been a variety of sclerosing agents u
throughout the years, but today the most common one is tetracycline. This app
work because of its low pH, and although the procedure is somewhat uncomfor
has a relatively high rate of success. For any sclerosing agent to work however,
must fully reexpand with drainage of the fluid. Failure to do so leaves an obligat
space, which will reaccumulate fluid with recurrence of symptoms. This situatio
treated with a pleural peritoneal shunt

(Q.170) Treatment of choice for medullary carcinoma of thyroid?

(a) Surgery only

(b) Radioablation by iodine

(c) Surgery and Radioablation by iodine

(d) Chemoradiation

Your Response :
Correct Answer : A

Exp: Surgery only (Ref. Bailey and Love 24 ed. 586)

MEDULLARY CARCINOMA

These are tumors of the parafollicular (C)-cells derived from the neural crest.

The cells are not unlike those of a carcinoid tumor and there is a characteristic a
stroma.

Diarrhoea is a feature in 30 per cent of cases and this may be due to 5-


hydroxytryptamine or pgs produced by the tumor cells.

High levels of serum calcitonin (>0.08 ng/ml) are produced by many medullary t

These levels fall after resection of a tumor and will rise again if the tumor recurs

This is a valuable tumor marker in the follow-up of patients with this disease.
Some tumors are familial and may account for 10—20 per cent of all cases.

Medullary carcinoma may occur in combination with adrenal pheochromocytom


hyperparathyroidism (usually due to hyperplasia) in the syndrome known as mu
endocrine neoplasia type ha (MEN IIa).

The familial form of the disease frequently affects children and young adults wh
the sporadic cases occur at any age with no sex predominance. When the famili
associated with prominent mucosal neuromas involving the lips, tongue and inn
aspect of the eyelids, with occasionally a Marfanoid habitus, the syndrome is re
as MEN type IIb.

Involvement of lymph nodes occurs in 50—60 per cent of cases of medullary ca


and blood-borne metastases are common.

As would be expected, tumors are not hormone dependent and do not take up
radioactive iodine.

The course of the tumor is unpredictable; in general, life expectancy is excellent


tumor is confined to the thyroid gland, good as long as metastases are confined
cervical lymph nodes and poor once blood-borne metastases are present.

Treatment is by total thyroidectomy and resection of involved lymph nodes with


radical or modified radical neck dissection.

(Q.171) A 5 year old male child has multiple hyper pigmented macules over the trunk, on rubbing the lesion
rounded end of a pen. He developed urticarial wheal, confined to the border of the lesion. The most
diagnosis is

(a) Fixed drug eruption

(b) Lichen planus

(c) Urticaria pigmentosa

(d) Urticarial vasculitis

Your Response :
Correct Answer : C

Exp: Urticaria pigmentosa

Urticaria is characterized by large, irregularly shaped pruritic, erythematous wh

Special forms of urticaria have special features (Dermographism, cholinergic urt


solar urticaria, or cold uritcaria)
Most incidents are acute and self-limited over a period of 1-2 weeks

Chronic urticaria (episodes lasting> 6 weeks) may have an autoimmune basis

The most common immunologic mechanism is hypersensitivity mediated by IgE


most patients with acute urticaria

The morphology of the lesions may vary over a period of minutes to hours, resu
geographic or bizarre, pattern, true urticaria last less than 24 hours and often o
hours.

The most common cause of acute urticaria are foods, viral infections and medic
Diagnosis - In vivo allergy skin testing and in vitro RAST testing

Determination of serum tryptase (Increased in anaphylaxis, systemic mastocyto


IgE mediated disease ("anaphylactoid reaction")

ACE inhibitors and Angiotensin II receptors antagonist therapy is contraindicate

Urticaria pigmentosa (mastocytosis) - consists of multiple irregularly shaped


hyperpigmented muscle which may present on the trunk and extremities.

Stroking the skin through the lesion with a blunt instrument elicits the classical "
response of Lewis" .

(Q.172) A 25 year old man presents with recurrent episodes of flexural eczema, recurrent skin infections and
abnormal. cramps and diarrhea upon taking sea foods. He is suffering from

(a) Seborrheic dermatitis

(b) Atopic dermatitis

(c) Airborne contact dermatitis

(d) Nummular dermatitis

Your Response :
Correct Answer : B

Exp: Atopic dermatitis

Clinical criteria for the diagnosis of Atopic dermatitis

1. Pruritus and scratching

2. Course marked by exacerbations and remissions


3. Lesions typical of eczematous dermatitis

4. Personal or family history of atopy (asthma, allergic rhinitis, food allergies or

5. Clinical course lasting longer than 6 weeks.

Diagnostic criteria
(I) A personal or family history of atopic disease (asthma allergic rhintis, atopic
dermatitis)

(II) Xerosis - ichthyosis

(III) Facial pallor with infraorbital darkening

(IV) Elevated serum IgE

(V) Fissures under the ear lobes

(VI) A tendency toward non specific hand dermatitis.

(VII) *A tendency towards repeated skin infections (staph aureus)

(VIII) A nipple eczema

Flexural lichenification in adults, pruritic, exudative or lichenified eruption on fa


upper trunk, wrists, and hands and in the antecubital and popliteal folds.

***Atopic dermatitis is diagnosed by clinical examination

The most common presentation of contact dennatitis is hand eczema

*The most common irritants encountered are chronic wetwork, soap and deter

Seborrhoeic dermatitis - This is an inflammation of the sebaceous glands which


manifests in the form of diffuse scaling and erythema on the scalp, forehead, th
eyebrows, margins of the eyelids, nasolabial folds, beard regions, infront of and
the external ear, chest, back axillae and groin.

(Q.173) Marjolin’s ulcer is a type of?

(a) Basal cell carcinoma

(b) Squamous cell carcinoma

(c) Malignant fibrous histiocytoma

(d) Neurotrophic malignant melanoma

Your Response :
Correct Answer : B

Exp: Squamous cell carcinoma REF : Manual on Clinical Surgeryby S.Das 6thEd P

Marjolin’s ulcer :
This is a squamous cell carcinoma arising from a long standing benign ulcer or sc

The commonest ulcer to become malignant is a long standing venous ulcer. It is


growing and less malignant form of squamous cell carcinoma. It differs from typ
squamous cell carcinoma in that its edges are not always raised and everted. Ly
metastasis are unusual. It is absolutely painless. It is radioresistant as it is relativ
avascular and there is extensive fibrosis.

(Q.174) Staphylococcal scalded skin syndrome (SSSS) may be characterized by all of the following statements
(a) It is associated with pharyngitis and conjunctivitis
(b) It occurs only in infants
(c) Bullons lesions result in a positive Nikolsky’s sign
(d) Once healed, there is usually no severe scarring
Your Response :
Correct Answer : B
Exp: It occurs only in infants
Staphylococcal scalded-skin syndrome (SSSS) is caused by a Staphylococcus aure
that produces an epidermolysin that results in a diffuse exfoliation. Although it
appears in young children, it can also occur in immunocompromised adults. SSS
limited to the upper epidermis.

(Q.175) Triplex DNA is formed because of

(a) In palindromic sequences

(b) Increase no. of guanosine repeat

(c) Increase in poly pyrimidine sequences

(d) Hoogstein pairing

Your Response :
Correct Answer : D

Exp: Hoogstein pairing

A Hoogsteen base pair is a variation of base-pairing in nucleic acids such as the A


pair. In this manner, two nucleobases on each strand can be held together by hy
bonds in the major groove.

A Hoogsteen base pair applies the N7 position of the purine base (as a hydrogen
acceptor) and C6 amino group (as a donor), which bind the Watson-Crick (N3–N
of the pyrimidine base.

This term is named for Karst Hoogsteen, who, in 1963, first recognized the pote
these unusual pairings.

Chemical properties

Hoogsteen pairs have quite different properties from Watson-Crick base pairs. T
between the two glycosylic bonds (ca. 80° in the A = T pair) is larger and the C1–
distance (ca. 860 pm or 8.6 Å) is smaller than in the regular geometry.

In some cases, called reversed Hoogsteen base pairs, one base is rotated 180° w
respect to the other.

Triplex structures

This non-Watson-Crick base-pairing allows the third strands to wind around the
duplexes, which are assembled in the Watson-Crick pattern, and form triple-stra
helices such as (poly(dA)•2poly(dT)) and (poly(rG)• 2poly(rC)). It can be also see
three-dimensional structures of transfer RNA.
To form a triplex, a sequence must conform to unusual requirements. One can s
the guanine and thymine residues in the half of the purine-rich strand that turn
make a triplex form Hoogsteen hydrogen bonds with guanine and adenine resid
respectively, in the other half of the same strand

References:

In vivo veritas: Using yeast to probe the biological functions of G-quadruplexes.


JE, Smith JS, Kozak ML, Johnson FB. Biochimie. 2008

(Q.176) All are true about oxygenase except

(a) Incorporate one atom of O2

(b) Incorporate both atoms of O2

(c) Hydroxylation of steroids

(d) Help in carboxylation of drugs

Your Response :
Correct Answer : D
Exp: Help in carboxylation of drugs

Ref: Harper Illustrated Biochemistry 28Ed ch 12 & lippincott - modern pharma


with clinical applications 6ed ch 23

In addition, molecular oxygen is incorporated into a variety of substrates by enz


designated as oxygenases; many drugs, pollutants, and chemical carcinogens
(xenobiotics) are metabolized by enzymes of this class, known as the cytochrom
system. Administration of oxygen can be lifesaving in the treatment of patients
respiratory or circulatory failure.
Oxygenases Catalyze the Direct Transfer & Incorporation of Oxygen into a Subst
Molecule.
Oxygenases are concerned with the synthesis or degradation of many different
metabolites. They catalyze the incorporation of oxygen into a substrate molecu
steps: (1) oxygen is bound to the enzyme at the active site, and (2) the bound ox
reduced or transferred to the substrate. Oxygenases may be divided into two su
as follows.
Dioxygenases Incorporate Both Atoms of Molecular Oxygen into the Substrate
The basic reaction is shown below:
Examples include the liver enzymes, homogentisate dioxygenase (oxidase) and
hydroxyanthranilate dioxygenase (oxidase), that contain iron; and L-tryptopha
dioxygenase (tryptophan pyrolase) that utilizes heme.
Monooxygenases (Mixed-Function Oxidases, Hydroxylases) Incorporate Only On
of Molecular Oxygen into the Substrate
The other oxygen atom is reduced to water, an additional electron donor or cos
(Z) being necessary for this purpose.
Cytochromes P450 Are Monooxygenases Important for the Detoxification of Ma
& for the Hydroxylation of Steroids
Cytochromes P450 are an important superfamily of heme-containing monooxge
and more than 1000 such enzymes are known. Both NADH and NADPH donate r
equivalents for the reduction of these cytochromes, which in turn are oxidized b
substrates in a series of enzymatic reactions collectively known as the hydroxyl
cycle.
In liver microsomes, cytochromes P450 are found together with cytochrome b5
have an important role in detoxification. Benzpyrene, aminopyrine, aniline, mor
and benzphetamine are hydroxylated, increasing their solubility and aiding their
excretion. Many drugs such as phenobarbital have the ability to induce the form
microsomal enzymes and of cytochromes P450.
(Q.177) Following are required for PCR except

(a) Deoxyribonucleotides

(b) Taq polymerase


(c) Dideoxyribonucleotides

(d) Template DNA

Your Response :
Correct Answer : C

Exp: Dideoxyribonucleotides

REF: Harper Illustrated Biochemistry 28Ed ch 39

Polymerase Chain Reaction

The polymerase chain reaction (PCR) is a test tube method for amplifying a sele
sequence that does not rely on the biologic cloning method described on p. 467
permits the synthesis of millions of copies of a specific nucleotide sequence in a
hours.
It can amplify the sequence, even when the targeted sequence makes up less th
part in a million of the total initial sample. The method can be used to amplify D
sequences from any source—bacterial, viral, plant, or animal.
A. Steps of a PCR

PCR uses DNA polymerase to repetitively amplify targeted portions of DNA. Eac
amplification doubles the amount of DNA in the sample, leading to an exponent
increase in DNA with repeated cycles of amplification. The amplified DNA seque
then be analyzed by gel electrophoresis, Southern hybridization, or direct seque
determination.

Primer construction: It is not necessary to know the nucleotide sequence of the


DNA in the PCR method. However, it is necessary to know the nucleotide seque
short segments on each side of the target DNA. These stretches, called flanking
sequences, bracket the DNA sequence of interest. The nucleotide sequences of
flanking regions are used to construct two, single-stranded oligonucleotides, us
35 nucleotides long, which are complementary to the respective flanking seque
The 3′-hydroxyl end of each primer points toward the target sequence (see Figu
33.19). These synthetic oligonucleotides function as primers in PCR reactions.

Denature the DNA: The DNA to be amplified is heated to separate the double-s
target DNA into single strands.

Annealing of primers to ssDNA: The separated strands are cooled and allowed
to the two primers (one for each strand).

Chain extension: DNA polymerase and deoxyribonucleoside triphosphates (in e


are added to the mixture to initiate the synthesis of two new chains complemen
the original DNA chains. DNA polymerase adds nucleotides to the 3′-hydroxyl en
primer, and strand growth extends across the target DNA, making complementa
copies of the target. [Note: PCR products can be several thousand base pairs lon
the completion of one cycle of replication, the reaction mixture is heated again
denature the DNA strands (of which there are now four). Each DNA strand binds
complementary primer, and the cycle of chain extension is repeated. By using a
stable DNA polymerase (for example, Taq polymerase) from a bacterium (for ex
Thermus aquaticus) that normally lives at high temperatures (a thermophilic ba
the polymerase is not denatured and, therefore, does not have to be added at e
successive cycle. Typically 20–30 cycles are run during this process, amplifying t
by a million-fold to a billion-fold. [Note: Each extension product of the primer in
sequence complementary to the primer at the 5′-end of the target sequence (se
33.19). Thus, each newly synthesized polynucleotide can act as a template for th
successive cycles (see Figure 33.20). This leads to an exponential increase in the
of target DNA with each cycle hence the name “polymerase chain reaction.”+

Dideoxyribonucleotides look exactly like deoxyribonucleotides except that the


group on the third carbon of the sugar.
This has very serious implications. These nucleotides can add to the growing DN
(because they have a phosphate on their fifth carbon that can link to the hydrox
third carbon of the nucleotide at the end of the growing chain) but once on, no
additional nucleotides can be added.
In other words, dideoxyribonucleotides act as chain terminators. When they ar
incorporated, the polymerization ends. It is the ability to end chains with these
molecules that is the basis for DNA sequencing by the Sanger procedure.

(Q.178) False about p 53

(a) Present on chr 17

(b) Arrest cell cycle in G1

(c) Wt 53 KDA prot

(d) Non mutated wild p53 is ass with neoplasms in childhood

Your Response :
Correct Answer : D

Exp: Non mutated wild p53 is ass with neoplasms in childhood

p53
p53, is atranscription factorwhich in humans is encoded by theTP53gene.
p53 is important inmulticellular organisms, where it regulates thecell cycleand t
functions as atumor suppressorthat is involved in preventingcancer.
As such, p53 has been described as "the guardian of thegenome," "the guardian
gene," and the "master watchman," referring to its role in conserving stabilit
preventing genome mutation.
The name p53 is in reference to its apparentmolecular mass: it runs as a
53kilodalton (kDa) protein onSDS-PAGE.
Gene: In humans, p53 is encoded by theTP53gene (guardian of the cell) located
short arm ofchromosome 17
Role in disease
If theTP53gene is damaged, tumor suppression is severely reduced. People who
only one functional copy of theTP53gene will most likely develop tumors in e
adulthood, a disease known asLi-Fraumeni syndrome.
TheTP53gene can also be damaged in cells bymutagens(chemicals,radiation,
orviruses), increasing the likelihood that the cell will begin decontrolled divis
More than 50 percent ofhuman tumorscontain amutationordeletionof
theTP53gene Increasing the amount of p53, which may initially seem a good wa
treat tumors or prevent them from spreading, is in actuality not a usable metho
treatment, since it can cause premature aging.
Certain pathogens can also affect the p53 protein that theTP53gene expresses.
example, theHuman papillomavirus(HPV), encodes a protein, E6, which binds th
protein and inactivates it.
Persistent infection over the years causes irreversible changes leading toCarcino
situand eventually invasive cervical cancer. This results from the effects of HP
particularly those encoding E6 and E7, which are the two viral onco proteins th
preferentially retained and expressed in cervical cancers by integration of the v
into the host genome.
In healthy humans, the p53 protein is continually produced and degraded in the
The degradation of the p53 protein is, as mentioned, associated with MDM2 bin
a negative feedback loop MDM2 is itself induced by the p53 protein. However m
p53 proteins often don't induce MDM2, and are thus able to accumulate at ver
concentrations. Worse, mutant p53 protein itself can inhibit normal p53 prot
levels
(Q.179) Which of the following statements is true with regard to patients who have paranoid personality dis

(a) They usually also suffer from paranoid psychosis

(b) They have a predisposition to develop schizophrenia

(c) They often have a preoccupation with helping the weak and the powerless

(d) They are often litigious

Your Response :
Correct Answer : D
Exp: They are often litigious

Persons with a paranoid personality disorder characteristically show marked


suspiciousness of others and are extremely sensitive to any potential threat or i
They frequently look for hidden motives or meanings, are contemptuous of the
and are very sensitive to issues of power and dominance. They often are morali
self-righteous and may be quite litigious. The percentage of affected persons wh
to develop schizophrenia is not known, but schizophrenia or paranoid psychosis
the typical outcome.

(Q.180) B cell marker are all except?

(a) Cd 19

(b) Cd 20

(c) Cd 10

(d) CD135

Your Response :
Correct Answer : D

Exp: CD135

CD135 is a proto-oncogene, meaning that mutations of this protein can lead to


Mutations of the Flt3 receptor can lead to the development of leukemia, a canc
bone marrow hematopoietic progenitors.
Internal tandem duplications of Flt3 (Flt3-ITD) are the most common mutations
associated with acute myelogenous leukemia (AML) and are a poor prognostic i
CD10 is also known as CALLA (common acute lymphocytic leukemia antigen. It s
a marker for the common form of ALL (acute lymphocytic leukemia) as well as fo
lymphoma and follicular germinal center lymphoma. CD10/CALLA is normally pr
the surface of early lymphoid cells as well as on a number of other types of norm
such as especially cells in the kidney
CD19 is expressed on follicular dendritic cells and B cells. In fact, it is present on
from earliest recognizable B-lineage cells during development to B-cell blasts bu
on maturation to plasma cells
CD20- MOST COMMONLY USED B-CELL MARKER in general pathology practise a
the world.
(Q.181) 15 year old female presents with primary amenorrhoe(A.) Her breasts are Tanner 4 but she has no a
pubic hair. The most likely diagnosis is:
(a) Turner’s syndrome

(b) Mullerian agenesis

(c) Testicular feminization syndrome

(d) Premature ovarian failure

Your Response :
Correct Answer : C

Exp: (Testicular feminization syndrome)

Testicular feminization syndrome [complete androgen insensitivity syndrome]:

It is a genetic disorder that makes XY fetuses insensitive (unresponsive)


to androgens (male hormones). Instead, they are born looking externally like no
girls. Internally, there is a short blind-pouch vagina and no uterus, fallopian tube
ovaries. There are testes in the abdomen or the inguinal canal.

The complete androgen insensitivity syndrome is usually detected at puberty w


should but does not begin to menstruate. Many of the girls with the syndrome h
pubic or axillary (armpit) hair. They have luxuriant scalp hair without temporal (
pattern) balding. They are sterile and cannot bear children.

They are at high risk for osteoporosis and so should take estrogen replacement

The gene for the syndrome is on the X chromosome in band Xq11-q12.

(Q.182) Most common cause of Conn’s syndrome?


(a) Cortical adenoma
(b) Adrenal hyperplasia
(c) Carcinoma
(d) Pheochromocttyoma
Your Response :
Correct Answer : A
Exp: Cortical adenoma
Aldosteronism - excess secretion of aldosterone - can be:

Primary - due to primary pathology of the adrenal gland

Secondary - due to reduced plasma volume and increased angiotensin productio


Secondary aldosteronism is due to cirrhosis, nephrotic syndrome or cardiac failu

Conn's syndrome is primary hyperaldosteronism due to:


Aldosterone producing adrenal cortical adenoma (50%)

Bilateral idiopathic adrenal hyperplasia - idiopathic hyperaldosteronism (40%)

Aldosterone secreting carcinoma

Usually occurs between 30 and 60 years

Conn's syndrome accounts for 1% of cases of hypertension

Hypertension often responds poorly to treatment

Biochemically there is usually a hypokalaemic alkalosis

Serum potassium may be normal

Investigations needed to Confirm primary hyperaldosteronism

(Q.183) True about hemochromatosis?

(a) Genetically heterogeneous

(b) More common in women than in men

(c) Cannot be treated with Phlebotomy

(d) Fully penetrant

Your Response :
Correct Answer : A

Exp: Genetically heterogeneous (Ref. Harrison's, 18th ed. 3162)

Hemochromatosis is a common inherited disorder of iron metabolism in which


inappropriate increase in intestinal iron absorption results in deposition of exce
amounts of iron in parenchymal cells with eventual tissue damage and impaired
function.

The term hemosiderosis is used to describe the presence of stainable iron in tiss
tissue iron must be quantified to assess body iron status accurately.

Hemochromatosis refers to progressive iron overload, potentially leading to fibr


organ failure.

Cirrhosis of the liver, diabetes mellitus, arthritis, cardiomyopathy, and


hypogonadotropic hypogonadism are the major clinical manifestations.

HFE-associated hemochromatosis is one of the most common genetic diseases,


its prevalence varies in different ethnic groups. It is most common in population
northern European extraction in whom approximately 1 in 10 persons are heter
carriers and 0.3 to 0.5% are homozygotes. However, expression of the disease i
modified by several factors, especially alcohol consumption and dietary iron int
blood loss associated with menstruation and pregnancy, and blood donation. Th
expression of the disease is 5 to 10 times more frequent in men than in women
70% of affected patients develop the first symptoms between ages 40 and 60. T
disease is rarely evident before age 20, although with family screening and perio
health examinations, asymptomatic subjects with iron overload can be identifie
including young menstruating women. Recent studies in European non-blood ba
populations have revealed that 30% or more of homozygous individuals do not
evidence of iron overload. Thus, thepenetrance of the mutation is variable.

The HFE gene involved in the most common form of hemochromatosis was clon
1996. A homozygous G: A mutation resulting in a cysteine to tyrosine substitutio
position 282 (C282Y) is the most common mutation.

The onset of clinical disease is usually after age 50 years—earlier in men than in
however, because of widespread liver biochemical testing and iron screening, th
diagnosis is usually made long before symptoms develop.

Early symptoms are nonspecific (eg, fatigue, arthralgias).

Later clinical manifestations include arthropathy, hepatomegaly and evidence o


insufficiency (late finding), skin pigmentation (combination of slate-gray due to
brown due to melanin, sometimes resulting in bronze color), cardiac enlargeme
or without heart failure or conduction defects, diabetes mellitus with its compli
and impotence in men.

Interestingly, population studies have shown an increased prevalence of liver di


but not of diabetes, arthritis, or heart disease in C282Y homozygotes.

(Q.184) ST Joseph aspirin therapy is essentially advised for all of the following EXCEPT:

(a) Rheumatic arthritis

(b) IUGR

(c) Post myocardial infarction

(d) Pre-eclampsia

Your Response :
Correct Answer : A
Exp: SLE

Ref: H-18th edition, Pg-2727


Low dose aspirin therapy is useful in treatment of:
Preeclampsia
IUGR and
Post AMI patients.
In preecclamptic woman, thromboxane is increased and prostaglandin E 2 and
prostacyclin are decreased, resulting in vasoconstriction and sensitivity to infuse
angiotensin II. Low dose aspirin as explained above enhances concentration of
vasodilating prostanoids and restores refractoriness to infused angiotensin II. Lo
aspirin and Dipyridamole are beneficial in reducing incidence of superimposed
Preeclampsia and growth restriction.

Early antiplatelet therapy with low dose aspirin may prevent uteroplacental thro
placental infarction, and idiopathic fetal growth retardation in women with a hi
recurrent severe fetal growth restriction.

Arthralgias, arthritis, myalgias, fever and mild serositis may improve on NSAIDs
Salicylates. However there appears no rationale of using low dose aspirin therap
same and it is beneficial in antiphospholipid antibody syndrome.

(Q.185) Antibodies specific to Anti-phospholipid antibody syndrome?

(a) Anti-nuclear antibodies

(b) Anti-glycoprotein 2B antibodies

(c) Anti-carbohydrate antibodies

(d) Anti-beta 1 microglobulin

Your Response :
Correct Answer : B

Exp: Anti-glycoprotein 2B antibodies.

Ref: H-18th edition, Pg-2736


Antiphospholipid syndrome develops in patients expressing antibodies to anioni
phospholipids, particularly BETA2 glycoprotein 1
APL may be idiopathic or secondary when associated with other disorders. The
aPL are the LA and the anticardiolipin antibody (ACL) occurring together in most
but also independently. The antibody specificity is actually to b2-glycoprotein 1
(b2GP1), a phospholipid membrane- associated protein. Rarely antibodies to
prothrombin co-exist and can cause hypoprothrombinaemia and bleeding. The
mechanism of thrombosis is not clear; APL may act against other vitamin K depe
proteins PC and PS, or possibly the autoimmune state may lead to endothelial d
and/or platelet activation.

(Q.186) Which of the following area is not affected in children with Autistic Spectrum Disorder?
(a) Repetitive and restricted interests and activities
(b) Socialization
(c) Language
(d) Motor abilities
Your Response :
Correct Answer : D
Exp: Motor abilities
The three main areas affected in children with Autistic Spectrum Disorder includ
Repetitive and restricted interests and activities, Socialization and Language.
Pervasive Developmental Disorders/Autism Spectrum Disorders

KEY FEATURES

AUTISM ASPERGER RETT CHILDHOOD PERVASIVE


SYNDROME SYNDROME DISINTEGRATIVE DEVELOPME
DISORDER DISORDER—
OTHERWISE
SPECIFIED

Delayed and Similar to Almost Clinically significant F eatures of


disordered autism always regression in skills other autism
communication except affects girls (language, social spectrum di
language skills, bowel, bladder but insufficie
skills Regression control, play motor diagnosis of
relatively in skills skills) before 10 yr of specifically
intact between 6 age
and 18 mo
Atypical social of age
interaction

Restricted Usually not Repetitive


range of congnitively hand
interests delayed movements

Onset before 3
yr of age

(Q.187) Which is a common comorbid condition with ADHD?


(a) Learning Disability
(b) Autism
(c) OCD
(d) All of above
Your Response :
Correct Answer : D
Exp: All of above (REF : Nelson 18th Ed Chap 31 )
Differential Diagnosis of Attention-Deficit/Hyperactivity Disorder (Including
Coexisting Disorders)

COEXISTING CONDITIONS WITH POSSIB


ATTENTION-DEFICIT/HYPERACTIVITY
DISORDER

DIMENSIONAL FACTORS PRESENTATION

Behaviors are within the spectrum of Oppositional-defiant disorder


normal

Behaviors are problematic, but fall Anxiety disorders


short of meeting the full criteria for
diagnosis Conduct disorder

Depressive disorders

Learning disorders

Language disorders

DIAGNOSES WITH ASSOCIATED ATTENT

PSYCHOSOCIAL DEFICIT/HYPERACTIVITY DISORDER


BEHAVIORS

Response to physical or sexual abuse Fragile X syndrome

Response to inappropriate parenting Fetal alcohol syndrome


practices

Response to parental psychopathology Pervasive developmental disorders(Au

Response to acculturation Obsessive–compulsive disorder

Response to inappropriate classroom Tourette syndrome


setting Attachment disorder

Psychosis or schizophrenia

Adjustment disorder with mixed emotio


conduct

MEDICAL NEUROLOGIC

Thyroid disorders (including general Auditory and visual processing disorder


resistance to thyroid hormone)
Seizure disorder

Heavy metal poisoning (including lead) Neurodegenerative disorder

Adverse effects of medications Post-traumatic head injury

Effects of abused substances Postencephalitic disorder

Sensory deficits (hearing and vision)

(Q.188) In which of the following condition, pseudo-convergent squint is seen:

(a) Wide inter-pupillary distance

(b) Abducent squint

(c) Broad epicanthus

(d) Thyrotoxicosis

Your Response :
Correct Answer : C

Exp: Broad epicanthus

Epicanthus is a semilunar fold of skin situated above and occasionally covering t


canthus. It is normal in Mongolian races. It is usually bilateral and gives the false
impression that the eyes are far apart having a convergent squint (pseudo-conv
squint).

(Q.189) False regarding cystoid macular edema is:

(a) Fluorescense angiography is diagnostic


(b) Visual prognosis is poor

(c) . May develop after cataract extraction

(d) . It is more common after intracapsular lens extraction

Your Response :
Correct Answer : B

Exp: Visual prognosis is poor

Macular edema is probably the most common complication of cataract extractio


occurs in 50% of cases. It is usually a transient condition and seldom causes any
significant problem. A small percentage of patients develop chronic cystoid mac
edema (Irvine-Gass syndrome) with diminished central vision 1-3 months after t
cataract extraction. This complication is more common following vitreous loss a
common after extracapsular extraction (intact capsule) as compared to intracap
extraction. Fluorescein angiography is diagnostic which typically shows "flower
pattern" due to leakage of dye from parafoveal capillaries. Most cases resolve
spontaneously within 6 months and require no specific treatment and in those r
cases which do not resolve and cause a significant impairment of central visions
vitrectomy usually promotes resolution.

(Q.190) Organism which penetrates intact corneal epithelium is:

(a) E. coli

(b) N. gonorrhoeae

(c) Staph. aureus

(d) Strept. pneumoniae

Your Response :
Correct Answer : B

Exp: N. gonorrhoeae Ref: Parson’s Diseases of the eye 20/e Page 226, 186

Purulent keratitis is almost always exogenous due to pyogenic organisms which


the cornea from without. Only organisms which are able to invade normal intac
epithelium of cornea are Neisseria gonorrhoeae and Corynebacterium diphther
Many other bacteria are capable of producing ulceration especially Streptococc
pneumoniae when the epithelium is already damaged.
(Q.191) When a pectus excavatum defect is corrected surgically, the patient postoperatively is likely to show

(a) Decreased oxygen consumption

(b) Decreased total lung capacity

(c) Increased cardiac output

(d) Increased maximal workload

Your Response :
Correct Answer : B

Exp: Decreased total lung capacity

Studies of patients' pre- and post correction of a pectus excavatum show no cha
workload or cardiac function. One group of observers found a decrease in total
capacity. There is no question that repair of the defect is appropriate in order to
a significant cosmetic problem.

(Q.192) In a patient with the burn wound extending into the superficial epidermis without involving the derm
would present with all of the following except

(a) Healing of the wound spontaneously without scar formation

(b) Erythema

(c) Blister formation

(d) Painful

Your Response :
Correct Answer : C

Exp: Blister formation.

Epidermal Burns (First-Degree)


As implied, these burns involve only the epidermis. They do not blister, but beco
erythematous because of dermal vasodilation, and are quite painful. Over 2 to 3
the erythema and pain subside. By about the fourth day, the injured epithelium
desquamates in the phenomenon of peeling, which is well known after sunburn
Superficial Partial-Thickness (Second-Degree)
Superficial partial-thickness burns include the upper layers of dermis, and
characteristically form blisters with fluid collection at the interface of the epider
dermis. Blistering may not occur until some hours after injury, and burns origina
appearing to be epidermal may subsequently be diagnosed as superficial partial
thickness burns after 12 to 24 hours. When blisters are removed, the wound is p
wet; currents of air passing over it cause pain. The wound is hypersensitive, and
burns blanch with pressure. If infection is prevented, superficial partial-thicknes
heal spontaneously in less than 3weeks, and do so without functional impairme
rarely cause hypertrophic scarring, but in pigmented individuals the healed burn
never completely match the color of the surrounding normal skin.
Deep Burns
Deep Partial-Thickness (Second-Degree)
Deep partial-thickness burns extend into the reticular layers of the dermis. They
blister, but the wound surface is usually a mottled pink-and-white color immedi
after the injury because of the varying blood supply to the dermis (white areas h
little to no blood flow and pink areas have some blood flow). The patient compl
discomfort rather than pain. When pressure is applied to the burn, capillary refi
slowly or may be absent. The wound is often less sensitive to pinprick than the
surrounding normal skin. By the second day, the wound may be white and is usu
fairly dry. If not excised and grafted, and if infection is prevented, these burns w
in 3 to 9 weeks, but invariably do so with considerable scar formation. Unless ac
physical therapy is continued throughout the healing process, joint function can
impaired, and hypertrophic scarring is common.
Full-Thickness (Third-Degree)
Full-thickness burns involve all layers of the dermis and can heal only by wound
contracture, epithelialization from the wound margin, or skin grafting. They app
white, cherry red, or black, and may or may not have deep blisters. Full-thicknes
are described as being leathery, firm, and depressed when compared with adjoi
normal skin, and they are insensate. The difference in depth between a deep pa
thickness burn and a full-thickness burn may be less than 1 mm. The clinical app
of full-thickness burns can resemble that of deep partial-thickness burns. They m
mottled in appearance, rarely blanch on pressure, and may have a dry, white
appearance. In some cases, the burn is translucent, with clotted vessels visible i
depths. Some full-thickness burns, particularly immersion scalds, have a red app
and initially may be confused with superficial partial-thickness burns. However,
be distinguished because they do not blanch with pressure. Full-thickness burns
a classic burn eschar, a structurally intact but dead and denatured dermis that if
situ over days and weeks, separates from the underlying viable tissue.
Fourth-Degree
Fourth-degree burns involve not only all layers of the skin, but also subcutaneou
and deeper structures. These burns almost always have a charred appearance, a
frequently only the cause of the burn gives a clue to the amount of underlying t
destruction. Electrical burns, contact burns, some immersion burns, and burns s
by patients who are unconscious at the time of burning may all be fourth-degre
(Q.193) A 30-year-old male has erectile dysfunction. Which of the following should be the first line of investi
evaluation of this patient?
(a) Sildenafil citrate test

(b) Cavernosometry

(c) Arteriography

(d) Intracavernous injection of papaverine

Your Response :
Correct Answer : D

Exp: Intracavernous injection of papaverine

Management of a patient with erectile dysfunction depends on the underlying c


which may be psychogenic, neurogenic or vascular. Differentiating among these
is often difficult even with complete history, physical examination, and endocrin
evaluation.

Intracavernous infection of papaverine, first introduced in 1984, is a useful diag


tool, both inexpensive and minimally invasive and is the “first-line test for evalu
penile blood flow”. This is also the most commonly performed diagnostic proce
erectile dysfunction.

It confirms or negates the suspicion of vasculogenic impotence.

(Q.194) CSF otorrhea occurs due to:

(a) Trauma to tympanic membrane

(b) Trauma to petrous temporal bone

(c) Trauma to cribriform plate

(d) Trauma to parietal bone

Your Response :
Correct Answer : B

Exp: Trauma to petrous temporal bone

CSF otorrhea is seen in the fracture of the petrous temporal bone. Systemic ant
which cross the blood brain barrier should be given.

(Q.195) A 5 year old boy has been diagnosed to have posterior superior refraction pocket cholesteatoma All
constitute part of the management EXCEPT
(a) Audiometry

(b) Mastoid exploration

(c) Tympanoplasty

(d) Myringoplasty

Your Response :
Correct Answer : D

Exp: [Myringoplasty] (Ref: Dhingra, 5th edition, Page 416, 417)

Myringoplasty- Operation consist of closing a 'central perforation' in the tympan


membrane, in the tubotympanic (safe) type of CSOM,

Modified radical mastoidectomy is the treatment of choice in Dangerous type (u


type of CSOM as presented by posterior superior retraction pocket cholesteatom
Tympanoplasty is done to eradicate disease in the middle ear and to reconstruc
mechanism

Myringoplasty (Type 1 tympanoplasty) is done for safe type of central perforatio


repair of perforation tympanic membrane with a graft

Audiometry is essential for pre-operative assessment and to confirm the degree


type of hearing loss.

(Q.196) Matching is done in case control study to eliminate:

(a) Confounding factor

(b) Bias

(c) Sampling error

(d) Relative risk

Your Response :
Correct Answer : A

Exp: Confounding Factor

A confounding factor is the one which is associated both with exposure and dise
is distributed unequally in study and control groups e.g., smoking is a confoundi
while studying etiological role of alcohol in esophageal cancer because smoking
associated with alcoholism and is also an independent risk factor for esophagea
Thus, the role of alcohol consumption in causing esophageal cancer can be dete
only if influence of smoking is neutralized by having matched controls in the stu

(Q.197) Claviceps fusiformis is active ingredient in which of the following toxin?


(a) Aflatoxin
(b) Sanguinarine
(c) Pyrazolidone
(d) Ergot alkaloid
Your Response :
Correct Answer : D
Exp: Ergot alkaloid (Ref. Text book of PSM by Park 20th/ 571)
Ergot is not a storage fungus, but a field fungus, e.g., Claviceps fusiformis. Symp
acute but rarely fatal and include nausea, repeated vomiting, giddiness and dro
extending peripheral gangrene due to vasoconstriction of capillaries have Ergot
grains can be easily removed by floating them in 20% salt water.

(Q.198) Which one of the following is associated with prolonged fasting (3 or more days)?

(a) Decreased lipolysis

(b) Increased urinary excretion of nitrogen

(c) Decreased gluconeogenesis

(d) Increased glucose utilization by the brain

Your Response :
Correct Answer : C

Exp: Decreased gluconeogenesis Ref: Harrison, 17th Edition, Page no 2305

With prolonged fasting of 3 days or more, gluconeogenesis is decreased partly d


increased ketogenesis and lipolysis. The increased availability of ketones and fat
as a source of fuel for brain cells decreases the demand by the brain for glucose
decreased gluconeogenesis is reflected in a nitro gen excretion level at or below
values. Insulin levels decrease beginning as early as 24 h after a meal.

(Q.199) During a voluntary movement, the Golgi tendon organ provides the central nervous system with info
about which of the following?

(a) The length of the muscle being moved.

(b) The velocity of the movement


(c) The blood flow to the muscle being moved

(d) The tension developed by the muscle being moved

Your Response :
Correct Answer : D

Exp: The tension developed by the muscle being moved Ref: Ganong, 22nd Edi
Page no 133 - 134

The Golgi tendon organ (GTO) is located in the tendon of skeletal muscles and t
is in series with the muscle. Each time the muscle contracts, the GTO organ is st
in proportion to the tension developed by the muscle. The lb afferent fibers (wh
innervate the GTO) produce a train of action potentials with a frequency that is
proportion to the deformation of the GTO. The muscle length and speed of shor
are sent to the CNS by Ia afferents that innervate the intrafusal fibers within mu
spindles.

(Q.200) Which of the following is the most important role of the gamma motor neurons?

(a) Stimulate skeletal muscle fibers to contract

(b) . Maintain Ia afferent activity during contraction of muscle

(c) Generate activity Ib afferent fibers

(d) Detect the length of resting skeletal muscle

Your Response :
Correct Answer : B

Exp: Maintain Ia afferent activity during contraction of muscle Ref: Ganong,


22nd Edition, Page no 135

The gamma motoneurons innervate the intrafusal fibers of the muscle spindles.
skeletal muscle contracts, the intrafusal muscle fiber becomes slack and the Ia a
stop firing. By stimulating the intrafusal muscle fibers during a contraction, the
motoneurons prevent the intrafusal muscle fibers from becoming slack and thu
maintain Ia firing during the contraction.

Powered By : Yo

Вам также может понравиться